29
Sample test questions Krok 2 Medicine ()

Krok 2 - Центр тестування при МОЗ УкраїниKrok 2 Medicine () Терапевтичний профiль 2 1. A 25-year-old woman has been suffering from diabetes

  • Upload
    others

  • View
    457

  • Download
    16

Embed Size (px)

Citation preview

Page 1: Krok 2 - Центр тестування при МОЗ УкраїниKrok 2 Medicine () Терапевтичний профiль 2 1. A 25-year-old woman has been suffering from diabetes

Sample test questions

Krok 2 Medicine

()

Page 2: Krok 2 - Центр тестування при МОЗ УкраїниKrok 2 Medicine () Терапевтичний профiль 2 1. A 25-year-old woman has been suffering from diabetes

Терапевтичний профiль 2

1. A 25-year-old woman has beensuffering from diabetes mellitus since shewas 9. She was admitted into the nephrologyunit with significant edemas of the face, arms,and legs. Blood pressure - 200/110 mm Hg,Hb- 90 g/L, blood creatinine - 850 mcmol/L,urine proteins - 1.0 g/L, leukocytes - 10-15 inthe vision field. Glomerular filtration rate -10 mL/min. What tactics should the doctorchoose?

A. Transfer into the hemodialysis unitB. Active conservative therapy for diabeticnephropathyC. DietotherapyD. Transfer into the endocrinology clinicE. Renal transplantation

2. A 59-year-old woman was brought into therheumatology unit. Extremely severe caseof scleroderma is suspected. Objectively shepresents with malnourishment, ”mask-like”face, and acro-osteolysis. Blood: erythrocytes- 2.2 · 109/L, erythrocyte sedimentation rate -40 mm/hour. Urine: elevated levels of freeoxyproline. Name one of the most likelypathogenetic links in this case:

A. Formation of antibodies to collagenB. Formation of antibodies to native DNAC. Formation of antibodies to bloodcorpusclesD. Formation of antibodies to transverselystriated musclesE. Formation of antibodies to vessel wall

3. A 34-year-old man on the 3rd day ofceftriaxone treatment for acute otitis (dailydosage - 2 grams) developed diarrheaoccurring 5-6 times per day. Feces are withoutmucus or blood admixtures. Temperature is36.6oC . Gregersen reaction (occult blood infeces) is negative. Stool culture detected nopathogenic germs. What is the most likelycause of diarrhea in this case?

A. Antibiotic-associated diarrheaB. Intestinal dysbiosisC. Bacterial overgrowth syndromeD. Ulcerative colitisE. Crohn’s disease (regional enteritis)

4. A chronic alcoholic was hospitalizedinto the therapeutic inpatient unit dueto pneumonia. On the day 5 of hishospitalization he became disoriented intime and space, developed fear-inducingvisual hallucinations and motor agitation.Full body tremor and tremor of the limbs areobserved. X-ray and physical examinationsdetect the signs of his convalescence frompneumonia. What tactics should be chosenregarding this patient?

A. Transfer into the inpatient narcologydepartmentB. Continue the treatment in the therapeuticdepartmentC. Transfer into the neuroresuscitationdepartmentD. Compulsory medical treatment foralcoholismE. Discharge from the hospital

5. After eating shrimps, a 25-year-old mansuddenly developed skin itching, some areasof his skin became hyperemic or erupted intovesicles. Make the diagnosis:

A. Acute urticariaB. Hemorrhagic vasculitis (Henoch-Schonleinpurpura)C. Urticaria pigmentosaD. PsoriasisE. Scabies

6. A 25-year-old woman complains of fatigue,dizziness, hemorrhagic rashes on the skin.She has been presenting with these signs for amonth. Blood test: erythrocytes - 1.0 · 1012/L,Hb- 37 g/L, color index - 1.1, leukocytes -1.2 ·109/L, platelets - 42 ·109/L. What analysiswould be the most advisable for diagnosis-making in this case?

A. Sternal puncture (bone marrow biopsy)B. Splenic biopsyC. Liver biopsyD. Coagulation studiesE. US of the gastrointestinal tract

7. A 35-year-old man complains of rapidlyincresing fatigue, palpitations, ”visual snow”,and dizziness. He has a history of pepticulcer of the stomach. Objectively the skinis pale. Vesicular respiration is observed inthe lungs. Systolic murmur is detected overthe cardiac apex, heart rate is 100/min., BPis 100/70 mm Hg. The epigastrium is slightlytender on palpation. Blood test: erythrocytes- 3.2 · 1012/L, Нb- 100 g/L, color index -0.95. What type of anemia is the most likelypresent in this case?

A. Posthemorrhagic anemiaB. Sideroblastic anemiaC. Chronic iron-deficiency anemiaD. Hemolytic anemiaE. Hypoplastic anemia

8. A 62-year-old patient has beenhospitalized with complaints of pain inthe thorax on the right during breathing,dyspnea, and dry cough. Ten days ago heslipped and fell hitting his right side. Onexamintaion: the patient lies on the leftside. The right side of the thorax lags duringbreathing. On the right there are crepitationand pain in the III-IV ribs. Dullness ofpercussion sound and sharply diminishedbreath sounds can be observed. On X-ray:

Page 3: Krok 2 - Центр тестування при МОЗ УкраїниKrok 2 Medicine () Терапевтичний профiль 2 1. A 25-year-old woman has been suffering from diabetes

Терапевтичний профiль 3

signs of exudate, fracture of the III-IV ribs.On pleurocentesis: blood is detected. Choosethe further tactics:

A. Transfer to a thoracic surgery departmentB. Prescribe conservative therapyC. Perform repeated pleural tapsD. Apply a fixation bandage to the rib cageE. Refer to a traumatologist

9. A 51-year-old man complains of vomitingwith blood. He has been drinking alcoholexcessively. Health disorder has beenobserved since he was 40, when he firstdeveloped jaundice. On examination theskin and visible mucosa are icteric, witha stellate vascular pattern. The patient ismalnourished and presents with abdominaldistension, umbilical hernia, and ascites. Theedge of the liver is tapered and painless, +3cm, the spleen is +2 cm. Blood test: Hb- 80g/L, leukocytes - 3·109/L, platelets - 85·109/L.What is the cause of portal hypertension inthis patient?

A. Hepatic cirrhosisB. Thrombosis of the splenic veinC. HemochromatosisD. Constrictive pericarditisE. Budd-Chiari syndrome

10. A young man has made an appointmentwith the dermatologist. He complains of apainful facial rash in the beard and mustachearea. This condition has been persistingfor several weaks already. After shaving,the patient’s condition aggravates. Thediagnosis of sycosis is made. What primarymorphological elements can be observed inthe rash in this case?

A. Pustules, papulaeB. Nodes, nodulesC. Pustules, bumpsD. Phlyctenae, maculaeE. Maculae, nodes

11. At night a 63-year-old woman suddenlydeveloped an asphyxia attack. She has a 15-year-long history of essential hypertensionand had a myocardial infarction 2 years ago.Objectively her position in bed is orthopneic,the skin is pale, the patient is covered withcold sweat, acrocyanosis is observed. Pulse- 104/min. Blood pressure - 210/130 mmHg, respiration rate - 38/min. Pulmonarypercussion sound is clear, with slight dullnessin the lower segments; throughout thelungs single dry crackles can be heard thatbecome bubbling and non-resonant in thelower segments. What is the most likelycomplication in this patient?

A. Acute left ventricular failureB. Paroxysmal tachycardiaC. Bronchial asthma attackD. Pulmonary embolismE. Acute right ventricular failure

12. A 26-year-old woman is suspected tosuffer from systemic lupus erythematosusdue to systemic lesions of skin, vessels, joints,serous tunics, and heart that developedafter photosensitization. The following isdetected: LE cells, antibodies to nativeDNA, isolated anti-centromere antibodies,rheumatoid factor is 1:100, Wassermannreaction is positive, circulating immunecomplex is 120 units. What immunologicalindicators are considered to be specific tothis disease?

A. DNA antibodiesB. Rheumatoid factorC. Anti-centromere antibodiesD. Immunoglobulin AE. Increased circulating immune complex

13. A woman came to the doctor withcomplaints of increased body temperature upto 37.8oC and moderately sore throat for thelast 3 days. Objectively: mandibular lymphnodes are enlarged up to 3 cm. Palatinetonsils are hypertrophied and covered withgray coating that spreads to the uvula andanterior pillars of the fauces. What is themost likely diagnosis?

A. Oropharyngeal diphtheriaB. Infectious mononucleosisC. Pseudomembranous (Vincent’s) tonsillitisD. AgranulocytosisE. Oropharyngeal candidiasis

14. A 42-year-old man, a dispatcher, suffesfrom peptic ulcer disease of the duodenum.The disease is of moderate severity. He wantsto be assigned a disability group. Make theconclusion regarding his working ability:

A. Capable of working, employableB. Capable of working, non-employableC. First group of disabilityD. Second group of disabilityE. Third group of disability

15. A 58-year-old man complains ofweakness and tumor-like formations thatappeared on the anterior surface of his neckand in the inguinal region. Palpation detectssoft painless mobile cervical and inguinallymph nodes up to 2 cm in diameter. Theliver protrudes by 2 cm from the edge of thecostal margin, the lower splenic pole is atthe umbilical level. In blood: erythrocytes- 3.5 · 1012/L, Hb- 88 g/L, leukocytes -86 · 109/L, band neutrophils - 1%, segmentedneutrophils - 10%, lymphocytes - 85%,eosinophils - 2%, basocytes - 0%, monocytes- 2%, erythrocyte sedimentation rate - 15

Page 4: Krok 2 - Центр тестування при МОЗ УкраїниKrok 2 Medicine () Терапевтичний профiль 2 1. A 25-year-old woman has been suffering from diabetes

Терапевтичний профiль 4

mm/hour, Gumprecht shadows. What is themost likely diagnosis?

A. Chronic lymphatic leukemiaB. Lymphocytic leukemoid reactionC. Acute leukemiaD. Chronic myeloleukemiaE. Lymphogranulomatosis

16. A 23-year-old man complains of facialedemas, headache, dizziness, low urinaryoutput, and urine discoloration (dark red).These complaints arose after a case of acutetonsillitis. On examination there are facialedemas, the skin is pale, temperature is37.4oC ; heart rate is 86/min., blood pressureis 170/110 mm Hg. Heart sounds are muffled,the II heart sound is accentuated over theaorta. What etiological factor is the mostlikely in this case?

A. Beta-hemolytic streptococcusB. Staphylococcus aureusC. Streptococcus viridansD. Streptococcus pyogenesE. Staphylococcus saprophyticus

17. An 18-year-old young man complains ofpain in his knee and elbow joints and bodytemperature up to 39.5oC . One week anda half earlier he developed sore throat. Onexamination his body temperature is 38.5oC .Swelling of the knee and elbow joints isobserved. Pulse is 106/min., rhythmic. Bloodpressure is 90/60 mm Hg. Cardiac bordersare unchanged, heart sounds are weakened,at the cardiac apex there is a soft systolicmurmur. What factor would be the mostindicative of the likely disease etiology?

A. Anti-streptolysin OB. C-reactive proteinC. Creatine kinaseD. Rheumatoid factorE. Seromucoid

18. A woman has been provisionallydiagnosed with pheochromocytoma. At thestage of intermission her BP is within norm;there is a tendency towards tachycardia. Nourine pathologies. The decision has beenmade to perform a provocative test withhistamine. What drug should be kept closeat hand for emergency aid in case of positivetest result?

A. PhentolamineB. Pipolphen (Promethazine)C. NifedipineD. Mesaton (Phenylephrine)E. Prednisolone

19. A 40-year-old man with Bekhterevdisease (ankylosing spondylitis) complains ofelevated body temperature up to 37.8oC , backpain and stiffness, especially observed duringthe second half of the night. This condition

has been lasting for 2 years. Objectively:reduced spinal mobility, painful sacroiliacjoint, erythrocyte sedimentation rate - 45mm/hour. X-ray shows narrowing of theintervertebral disc space and of the sacroiliacjoint. What eye pathology is often associatedwith this type of disease progression?

A. IridocyclitisB. Retinal detachmentC. CataractD. Optic nerve atrophyE. Blepharitis

20. A 63-year-old man complains ofunmotivated weakness and pressing andbursting sensation in the left subcostalarea. According to him, these signshave been present for a year already.Previously he was healthy. He took part incontainment measures during the accidentat the Chornobyl Nuclear Power Plant.Objectively: the skin is pale, peripherallymph nodes are not enlarged, the liveris +3 cm, the spleen is +10 cm. Completeblood count: erythrocytes - 3.1 · 1012/L,Hb- 100 g/L, leukocytes - 46 · 109/L, blasts- 2%, promyelocytes - 10%, myelocytes -18%, band neutrophils - 27%, segmentedneutrophils - 10%, lymphocytes - 12%,eosinophils - 6%, basocytes - 3%, monocytes- 2%, erythrocyte sedimentation rate - 20mm/hour. What is the most likely diagnosis?

A. Chronic myeloleukemiaB. Hepatic cirrhosisC. Acute leukemiaD. Hemolytic anemiaE. Chronic lymphatic leukemia

21. For three years a 31-year-old woman hasbeen complaining of pain and swelling ofher radiocarpal and metacarpophalangealarticulations and their reduced mobility inthe morning, which persisted up to 1.5 hours.Two weeks ago she developed pain, swelling,and reddening of her knee joints, her bodytemperature increased up to 37.5oC . Thetreatment was untimely. Examination ofthe internal organs revealed no pathologicalterations. Diagnosis of rheumatoid arthritiswas made. What changes are most likely tobe visible on the arthrogram?

A. Joint space narrowing, usurationB. Joint space narrowing, subchondralosteosclerosisC. Cysts in the subchondral boneD. Numerous marginal osteophytesE. Epiphyseal osteolysis

22. A 52-year-old woman has beensuffering for 2 years from dull, occasionallyexacerbating pain in her right subcostalarea, occurring after eating high-fat foods,bitter taste in her mouth in the morning,constipations, and flatulence. Objectively she

Page 5: Krok 2 - Центр тестування при МОЗ УкраїниKrok 2 Medicine () Терапевтичний профiль 2 1. A 25-year-old woman has been suffering from diabetes

Терапевтичний профiль 5

has excess weight, her body temperatureis 36.9oC ; there is a coating on the rootof her tongue; the abdomen is moderatelydistended and painful in the area ofgallbladder projection. What examinationwould be the most helpful for diagnosis-making?

A. UltrasoundB. Duodenal intubationC. CholecystographyD. DuodenoscopyE. Liver scanning

23. A 57-year-old woman complains ofweakness, dyspnea, loss of appetite, andliquid feces. She has been suffering fromthis condition for 2 years. Objectivelyshe presents with pale skin, subictericsclerae, and bright-red fissured tongue.Lymph nodes are not enlarged. Pulse- 100/min. BP- 105/70 mm Hg. Liver+3 cm, the spleen cannot be palpated.Blood test: erythrocytes - 1.2 · 1012/L,Нb- 56 g/L, color index - 1.4, macrocytes,leukocytes - 2, 5 · 109/L, eosinophils - 1%,juvenile - 1%, metamyelocytes - 1%, bandneutrophils - 8%, segmented neutrophils -47%, lymphocytes - 38%, monocytes - 4%,reticulocytes - 0.1%, platelets - 100 · 109/L,ESR- 30 mm/hour, indirect bilirubin - 26mmol/L. What changes can be expected inthe bone marrow puncture material?

A. Prevalence of megaloblastsB. Increased number of sideroblastsC. Erythroid hyperplasiaD. Presence of blast cellsE. Prevalence of lymphoid tissue

24. A 35-year-old man suffers from insulin-dependent diabetes mellitus and chroniccholecystitis. He takes NPH insulin: 20 unitsin the morning and 12 units in the evening.After a meal he developed pain in the rightsubcostal area, nausea, vomiting, sleepiness,and increased polyuria. What prehospitalmeasures will be the most effective forprevention of crisis within the next severalhours?

A. Change insulin regimenB. Take analgesicsC. Take cholagoguesD. Exclude fats from the dietE. Decrease carbohydrates in the diet

25. A 45-year-old woman complains ofintolerable paroxysmal facial pain on the leftwith attacks that last for 1-2 minutes. Attacksare provoked by chewing. The disease onsetwas two months ago after the overexposureto cold. Objectively: pain at the exit pointsof the trigeminal nerve on the left. Touchingnear the wing of the nose on the left inducesa pain attack with tonic spasm of the facial

muscles. What is the most likely diagnosis?

A. Trigeminal neuralgiaB. Glossopharyngeal neuralgiaC. Temporomandibular joint arthritisD. Facial migraineE. Maxillary sinusitis

26. A 28-year-old man complains of skinrash and itching on the both of his hands.The condition persists for 1.5 years. Theexacerbation of his condition he ascribes tothe occupational contact with formaldehyderesins. Objectively the lesion foci aresymmetrically localized on both hands.Against the background of erythema withblurred margins there are papulae, vesicles,erosions, crusts, and scales. What is the mostlikely pathology?

A. Occupational eczemaB. Idiopathic eczemaC. Allergic dermatitisD. Simple contact dermatitisE. Erythema multiforme

27. A 20-year-old student after failing anexam developed complaints of a sensation ofa round foreign body in her throat, difficultswallowing. She fixates on her condition,limits her diet, often cries, seeks attention,exhibits demonstrative attitude. She is highlysusceptible to psychotherapeutic suggestion.What psychiatric diagnosis can be made inthis case?

A. Hysterical neurosisB. Hypochondriacal neurosisC. Depressive neurosisD. Obsessive neurosisE. Paranoid personality disorder

28. A woman with atopic bronchial asthmawas found to have one allergen to doghair +++. Carpets were removed from theapartment, the apartment was renovated,and air conditioner was installed. However,recurrent asphyxia attacks still occur everynight, despite the patient undergoingpathogenetic therapy. What long-termtreatment tactics can help this patient todecrease her sensitivity to the allergen?

A. Specific hyposensitizationB. Continuation of prior treatmentC. Antihistamine therapyD. Buteyko breathing techniqueE. Referral for speleotherapy

29. A 20-year-old man was hospitalized onthe 9th day of the disease. He attributeshis disease to eating of insufficientlythermally processed pork. At its onsetthis condition manifested as periorbitaledemas and fever. Objectively his bodytemperature is 38.5�C . The face is puffy andthe eyelids are markedly swollen. Palpation

Page 6: Krok 2 - Центр тестування при МОЗ УкраїниKrok 2 Medicine () Терапевтичний профiль 2 1. A 25-year-old woman has been suffering from diabetes

Терапевтичний профiль 6

of gastrocnemius muscles is sharply painful.Blood test shows hypereosinophilia. What isthe etiology of this disease?

A. TrichinellaB. TrichurisC. AscarididaeD. EchinococciE. Leptospira

30. A 40-year-old man claims that his wifeis cheating on him and presents a ”proof”of her infidelity. He repeatedly initiatedscandals with his wife at home and at work,demanding that she confess her infidelity,insulted her, and threatened to kill her. Whatpreventive measures should be taken againstsocially dangerous actions on his part?

A. Consultation with the psychiatristB. Outpatient treatmentC. Consultation with the general practitionerD. Consultation with the psychologistE. Family counseling

31. A 55-year-old woman complains ofpain and popping sounds in her left kneejoint, which occur when she climbs thestairs. Occasionally during movements herjoint becomes ”stuck”. 5 years ago shesuffered a trauma of her left knee. Completeblood count and biochemical blood analysisshow normal results. X-ray shows markedosteosclerosis and osteophytes. The jointspace is narrowed. Make the provisionaldiagnosis:

A. OsteoarthritisB. Rheumatoid arthritisC. Gouty arthritisD. Psoriatic arthritisE. Reactive arthritis

32. A 40-year-old man, a welder, usesmanganese electrodes in his line of work(18 years of experience). He complains ofdifficulties with writing, bad mood, inertness,gait abnormalities, problems with speech,and hand tremors. Objectively the followingis observed in the patient: hypomimia,increased muscle tone of plastic type, andquiet monotonous speech, tremor of thetongue, pill-rolling tremor of the fingers, andretropulsion. What syndrome developed inthis patient due to manganese poisoning?

A. ParkinsonismB. MeningismC. Hypothalamic syndromeD. Polyneuritic syndromeE. Vestibular syndrome

33. A 45-year-old man developedconstricting retrosternal pain that occursduring walks at the distance of 200 m.Objectively: heart rate is 80/min., BP is160/90 mm Hg. During cardiopulmonary

exercise test at 50 W there is a depressionof S-T segment by 3 mm below the isoline inV3-V4. What is the provisional diagnosis?

A. Exertional angina pectoris, functional classIIIB. Exertional angina pectoris, functional classIVC. Exertional angina pectoris, functional classIID. Somatoform autonomic dysfunction,hypertension typeE. Alcoholic myocardiodystrophy

34. A 45-year-old man, a farmer, presentswith acute onset of a disease. He complainsof headache, high temperature, pain in thegastrocnemius muscles, icteric face, and darkurine. Objectively: body temperature - 38oC ,blood pressure - 100/70 mm Hg, conjunctivalhemorrhages, hepatosplenomegaly, andoliguria. What is the most likely provisionaldiagnosis?

A. LeptospirosisB. BrucellosisC. Viral hepatitisD. PseudotuberculosisE. Trichinosis

35. A woman has been working as a polisherfor a year and a half. Her workstationis equipped with a grinding machine(grinding wheels). She complains of whitediscoloration of her fingers and toes thatappears when she is nervous. Objectivelythere are no changes in the coloration of thedistal segments of her limbs. Grip strengthmeasured with a dynamometer is 25 kg,algesimetry findings are 0.1; 0.3; 0.5. Coldstimulus is extremely positive on the upperand lower limbs. Internal organs are withoutpathologies. Make the diagnosis:

A. Vibration diseaseB. Raynaud diseaseC. SyringomyeliaD. Raynaud syndromeE. Polyneuritis

36. A 37-year-old man suddenly developedacute headache accompanied by nausea,vomiting, and impaired consciousness.Objectively blood pressure is 190/120mm Hg, the face is hyperemic. Patient’sconsciousness is clouded, his answers tothe questions are short, monosyllabic.Movement and sensory disturbances areabsent. Meningeal signs are positive.Cerebrospinal fluid contains blood. Whatprovisional diagnosis can be made?

Page 7: Krok 2 - Центр тестування при МОЗ УкраїниKrok 2 Medicine () Терапевтичний профiль 2 1. A 25-year-old woman has been suffering from diabetes

Терапевтичний профiль 7

A. Subarachnoid hemorrhageB. MeningitisC. Ischemic strokeD. EncephalitisE. Cerebral vascular embolism

37. A woman undergoing in-patienttreatment for viral hepatitis type Bdeveloped headache, nausea, recurrentvomiting, memory lapses, flapping tremorof her hands, and rapid pulse. Sweetsmell from her mouth is detected. Bodytemperature is 37.6oC , heart rate is 89/min.What complication developed in the patient?

A. Acute liver failureB. Ischemic strokeC. Gastrointestinal hemorrhageD. Hypoglycemic shockE. Meningoencephalitis

38. A 43-year-old man, a coal-face workerwith 15-year-long record of work, complainsof cough, thoracic pain, and dyspnea. Thecough is mild, usually dry, occurs mostlyin the morning. The pain is localized inthe interscapular region and aggravatesduring a deep intake of breath. Dyspneaoccurs during physical exertion. Vesicularrespiration in the lungs is weakened. Heartsounds are rhythmic, heart rate is 86/min.,blood pressure is 135/80 mm Hg. Theabdomen is soft and painless. X-ray showsmicronodular pulmonary fibrosis. Make theprovisional diagnosis:

A. CarboconiosisB. ByssinosisC. SiderosisD. BerylliosisE. Metal pneumoconiosis

39. Having examined a 52-year-old patient,the doctor diagnosed him with obesity (bodymass index - 34 kg/m2, waist circumference- 112 cm) and arterial hypertension (170/105mm Hg). 2-hour postprandial blood sugaris 10.8 mmol/L. What biochemical bloodanalysis needs to be conducted to diagnosethe patient with metabolic syndrome X?

A. Lipid profileB. BilirubinC. Calcium and phosphorusD. Creatinine and ureaE. Electrolytes

40. After overexposure to cold a 45-year-old woman developed acute pain inher suprapubic and lumbar areas duringurination, sharp pains at the end of urination,false urges to urinate. Urine is turbid withblood streaks. The doctor suspects urinarytract infection. What results of laboratoryanalysis would be the most indicative of suchinfection?

A. Leukocyturia, gross hematuriaB. Gross hematuriaC. Increased blood creatinine and blood ureaD. Daily proteinuria under 3.0E. Daily proteinuria over 3.0

41. A 62-year-old woman was broughtinto the admission room with complaintsof severe burning retrosternal pain andasphyxia. She has a 10-year-long historyof essential hypertension. Objectively hercondition is moderately severe. She presentswith skin pallor, cyanotic lips, and vesicularrespiration over her lungs. The II heart soundis accentuated over the aorta. Blood pressure- 210/120 mm Hg, heart rate (pulse) - 76/min.ECG shows elevation of ST segment in theleads I, AVL, and V5-V6. What is the mostlikely diagnosis?

A. Hypertensive crisis complicated with acutemyocardial infarctionB. Uncomplicated hypertensive crisisC. Hypertensive crisis complicated withinstable angina pectorisD. Hypertensive crisis complicated with acuteleft ventricular failureE. Pulmonary embolism

42. A 35-year-old patient developed anepileptic attack with tonoclonic spasms thatlasted for 3 minutes. After the attack thepatient fell asleep but in 5 minutes the secondattack occurred. The first step of emergencyaid would be to:

A. Ensure patency of airwaysB. Take blood from the vein for analysisC. Introduce diazepam intravenouslyD. Prescribe antiepileptic drugsE. Administer chloral hydrate via an enema

43. A 27-year-old woman, a teacher in theelementary school, complains of frequentstools, up to 3 times per day, with lumpyfeces and large amount of mucus, abdominalpain that gradually abates after a defecation,irritability. Her skin is pale and icteric. Pulseis 74/min., rhythmic, can be characterizedas satisfactory. Blood pressure is 115/70 mmHg. The abdomen is soft, moderately tenderalong the colon on palpation. Fiberopticcolonoscopy detects no changes. Whatdisease can be suspected?

A. Irritable bowel syndromeB. Chronic non-ulcerative colitisC. Chronic enteritisD. Crohn disease (regional enteritis)E. Whipple disease

44. A 72-year-old man complains of lowerextremity edema, sensation of heaviness inthe right subcostal area, dyspnea at rest.For over 25 years he has been sufferingfrom COPD. Objectively: orthopnea,jugular venous distention, diffuse cyanosis,

Page 8: Krok 2 - Центр тестування при МОЗ УкраїниKrok 2 Medicine () Терапевтичний профiль 2 1. A 25-year-old woman has been suffering from diabetes

Терапевтичний профiль 8

acrocyanosis. Barrel chest is observed, onpercussion there is a vesiculotympanitic(bandbox) resonance, sharply weakenedvesicular respiration on both sides, moistcrepitant crackles in the lower segmentsof the lungs. Heart sounds are weakened,the II heart sound is accentuated over thepulmonary artery. The liver is +3 cm. Whatcomplicated the clinical course of COPD inthis patient?

A. Chronic pulmonary heartB. Pulmonary embolismC. Acute left ventricular failureD. Diffuse pneumosclerosisE. Community-acquired pneumonia

45. A 72-year-old man with pneumoniacomplains of marked dyspnea, chest pain,severe cough with expectoration, to is39.5-40oC , no urination for a wholeday. Objectively the patient is conscious.Respiratory rate is 36/min. Over the rightlower pulmonary lobe percussion soundis dull; on auscultation there is bronchialrespiration and numerous moist crackles.Blood pressure is 80/60 mm Hg. Heart rate is120/min. Heart sounds are muffled, there istachycardia. What tactics should the familydoctor choose in the management of thispatient?

A. Hospitalization into the intensive care unitB. Outpatient treatmentC. Treatment in the day patient facilityD. Hospitalization into the pulmonology unitE. Hospitalization into the neurology unit

46. 2 hours after eating unknownmushrooms, a 28-year-old man sensed adecrease in his mobility and deterioration ofhis ability to focus. This condition was thenfollowed by a state of agitation and agression.On examiantion he is disoriented and hisspeech is illegible. 4 hours later he developedfetor hepaticus and lost his consciousness.What syndrome can be observed in thispatient?

A. Acute hepatic failureB. Hepatolienal syndromeC. Portal hypertensionD. Cholestatic syndromeE. Cytolytic syndrome

47. A 36-year-old man complains of markeddyspnea and cardiac pain. He ascribes hisdisease to the case of influenza that he had2 weeks ago. Objectively he leans forwardwhen sitting. The face is swollen, cyanotic,cervical veins are distended. Heart bordersare extended on the both sides, heart soundsare muffled, heart rate = Ps = 118/min.,BP is 90/60 mm Hg. Blood test: ESR is46 mm/hour. ECG shows low voltage. X-ray shows trapezoidal cardiac silhouette andsigns of pulmonary congestion. Choose the

treatment tactics:

A. Pericardial puncture (pericardiocenthesis)B. DiureticsC. AntibioticsD. PericardectomyE. Glucocorticosteroids

48. A 39-year-old man suffers from chronicrheumatic heart disease. He complains ofdyspnea during physical exertion, cough withexpectoration, and palpitations. Ausculationdetects intensified I heart sound and diastolicmurmur; the sound of opening mitral valvecan be auscultated at the cardiac apex.The II heart sound is accentuated over thepulmonary artery. The patient is cyanotic.X-ray shows dilated pulmonary root andenlargement of the right ventricle and leftatrium. What is the most likely diagnosis?

A. Mitral stenosisB. Aortic stenosisC. Pulmonary artery stenosisD. Coarctation of the aortaE. Patent ductus arteriosus

49. A 23-year-old man complains of severepain in his left knee joint. Objectively theleft knee joint is enlarged, with hyperemicskin, painful on palpation. Complete bloodcount: erythrocytes - 3.8 · 1012/L, Hb- 122g/L, leukocytes - 7.4 · 109/L, platelets -183 · 109/L. Erythrocyte sedimentation rate- 10 mm/hour. Bleeding time (Duke method)- 4 min., Lee-White coagulation time - 24min. Partial thromboplastin time (activated)- 89 seconds. Rheumatoid factor - negative.What is the most likely diagnosis?

A. Hemophilia, hemarthrosisB. Werlhof disease (immunethrombocytopenia)C. Rheumatoid arthritisD. ThrombocytopathyE. Hemorrhagic vasculitis (Henoch-Schonleinpurpura), articular form

50. A 24-year-old woman, a kindergartenteacher, has been sick for 2 days already.Disease onset was acute. She presents withelevated body temperature up to 38.0oC ,pain attacks in her lower left abdomen, liquidstool in small amounts with blood and mucusadmixtures 10 times a day. Pulse - 98/min.,blood pressure - 110/70 mm Hg. Her tongueis moist and coated with white deposits. Theabdomen is soft, the sigmoid colon is painfuland spastic. Make the provisional diagnosis:

A. ShigellosisB. EscherichiosisC. SalmonellosisD. YersiniosisE. Rotavirus infection

Page 9: Krok 2 - Центр тестування при МОЗ УкраїниKrok 2 Medicine () Терапевтичний профiль 2 1. A 25-year-old woman has been suffering from diabetes

Терапевтичний профiль 9

51. A 38-year-old woman complains ofweakness, sleepiness, pain in the joints,weight gain despite low appetite, andconstipations. She presents with dry andthickened skin, puffy and amimic face,narrowed palpebral fissures, thick tongue,and deep hoarse voice. Her heart sounds areweak, pulse is 56/min. Low levels of free T4are observed. This patient needs to take thefollowing on a regular basis:

A. ThyroxineB. Mercazolil (Thiamazole)C. Lithium carbonateD. FurosemideE. Calcium gluconate

52. A 23-year-old man has accidentallyswallowed brake fluid. After that he has beenpresenting with anuria for 5 days already;his creatinine levels elevated up to 0.569mmol/L. What treatment tactics should bechosen in this case?

A. HemodialysisB. Detoxication therapyC. Antidotal therapyD. DiureticsE. Plasmapheresis

53. A 52-year-old man for the last 3 yearshas been suffering from difficult swallowingof solid food, burning retrosternal painthat aggravated during eating, loss ofbody mass, and occasional vomiting withundigested food. Esophageal X-ray showsS-shaped deformation of the esophagusand its dilation; at the cardiac orifice theesophagus is constricted; esophageal mucosais smooth, without signs of peristalsis. Makethe provisional diagnosis:

A. Esophageal carcinomaB. Diaphragmatic herniaC. Esophageal achalasiaD. Reflux esophagitisE. Esophageal diverticulum

54. A 53-year-old man complains ofgeneral weakness, loss of appetite, andpainful vesicles appearing on his skin.The disease onset occurred suddenly, afterhyperinsolation one week ago. Examinationdetects isolated vesicles with wrinkledopercula and occasional painful erosions onthe skin of the patient’s torso and limbs.Nikolsky sign is positive. What is the mostlikely diagnosis?

A. Acantholytic pemphigusB. Nonacantholytic pemphigusC. Duhring’s disease (dermatitisherpetiformis)D. HerpesE. Toxicodermia

55. A patient is being treated in the

tuberculosis clinic. Throughout the last 3weeks he has been suffering from headachesof increasing intensity. Neurologicalexamination detects nuchal rigidity withoutfocal signs. Make the provisional diagnosis:

A. Tuberculous meningitisB. Chorea minorC. Brain tumorD. MyelitisE. Convexital arachnoiditis

56. A patient has gradually lostconsciousness. The skin is pale and dry. Thereis a smell of ammonia from the mouth.Respirations are deep and noisy. Heartsounds are muffled, pericardial friction rubis present. Blood pressure is 180/130 mmHg. Blood test: Нb- 80 g/L, leukocytes -12 · 109/L, blood glucose - 6.4 mmol/L, urea -50 mmol/L, creatinine - 1200 mcmol/L, bloodosmolarity - 350 mOsmol/L. No urinaryexcretion. Make the diagnosis:

A. Uremic comaB. Hyperglycemic comaC. Acute renal failureD. Acute disturbance of cerebral circulationE. Hyperosmolar coma

57. A 72-year-old man diagnosed withischemic heart disease presents with diffusecardiosclerosis, permanent tachysystolicatrial fibrillation, heart failure IIа, FC III.Objective examination of vital signs: bloodpressure is 135/80 mm Hg, heart rate is160/min., pulse is 125/min. Left ventricularejection fraction is 32%. What drug isindicated in this case and should be presribedto the patient?

A. DigoxinB. Procainamide (Novocainamide)C. Isadrine (Isoprenaline)D. VerapamilE. Ivabradine

58. A 34-year-old man complains of paleedema of the face, feet, shins, and lumbararea, elevated blood pressure up to 160/100mm Hg, and general weakness. He hasa clinical history of nonspecific ulcerativecolitis. Objectively: pulse - 84/min., rhythmic,blood pressure - 165/100 mm Hg; edemas allover the body; the skin is pale and dry, withlow turgor. The kidneys cannot be palpated,on an attempt to palpate them they arepainless. Blood test: erythrocytes - 3.0·1012/L,Нb- 100 g/L, erythrocyte sedimentationrate - 50 mm/hour. Urinalysis: proteins -3.5 g/L, erythrocytes - 7-10 in the visionfield, leukocytes - 5-6 in the vision field.Daily proteinuria - 6 grams. What analysisshould be conducted additionally to verifythe diagnosis?

Page 10: Krok 2 - Центр тестування при МОЗ УкраїниKrok 2 Medicine () Терапевтичний профiль 2 1. A 25-year-old woman has been suffering from diabetes

Терапевтичний профiль 10

A. Gingival biopsy for the diagnosis ofamyloid diseaseB. Radioisotopic examination of kidneysC. Urinalysis for Bence-Jones proteinD. Renal ultrasoundE. Survey and excretory urography

59. A 42-year-old man, a worker at themeat processing factory, developed anitching spot on his lower jaw, whichgradually transformed into a slightly painfulcarbuncle 3 cm in diameter, surrounded bya painless swelling that reaches the clavicle.Temperature is subfebrile, under 37.8oC . Thedoctor suspects anthrax. What drug shouldthis man be prescribed for treatment?

A. PenicillinB. Levomycetin (Chloramphenicol)C. Biseptol (Co-trimoxazole)D. Interferon alphaE. Azidothymidin (Zidovudine)

60. A 57-year-old patient complains ofdyspnea at rest. The patient presents withorthopnea, acrocyanosis, bulging cervicalveins. On percussion: dull sound over thelower lung segments. On auscultation: norespiratory sounds. Heart rate is 92/min.Right-sided cardiac dilatation is observed.The liver is +7 cm. Shins are swollen. Pleuraleffusion is suspected. What indicator wouldconfirm the presence of transudate in thiscase?

A. Total protein content in the pleural fluidbelow 25 g/LB. Presence of atypical cellsC. Total protein content in the pleural fluidexceeding 30 g/LD. Specific gravity exceeding 1015E. Positive Rivalta’s test

61. A 33-year-old man developed multiplerashes on the skin of his torso and extensorsurfaces of his upper and lower limbs.The rashes itch and occasionally fusetogether and form plaques. The elementsof rash are covered with silver-white finescales that easily flake off when scratched.Grattage test results in three sequentialphenomena: stearin spot, terminal film, andpunctate hemorrhage. What diagnosis can besuspected?

A. PsoriasisB. ParapsoriasisC. PyodermaD. Lichen ruber planusE. Secondary papular syphilid

62. A 38-year-old woman after physicaloverexertion suddenly developed palpitations,dyspnea, and a dull pain in the cardiacarea. For 10 years she has been registeredfor regular check-ups due to rheumatismand mitral valve disease with non-disturbed

blood circulation. Oblectively her pulse is96/min., of unequal strength. Blood pressureis 110/70 mm Hg, heart rate is 120/min. ECGregisters small unevenly-sized waves in placeof P-waves, R-R intervals are of unequallength. What is the most likely diagnosis?

A. Atrial fibrillationB. Paroxysmal supraventricular tachycardiaC. Atrial flutterD. Paroxysmal ventricular tachycardiaE. Respiratory arrhythmia

63. An 18-year-old patient always obeysothers and adapts his needs to the demandsof the people on whom he depends. Heexcessively defers to their wishes and makesthem responsible for his wellbeing, cannotdefend his interests and needs supportfrom other people. Such psychic profilehas been formed in the childhood, remainsunchanged, and hinders adaptation. Whatpsychic disorder is observed in this patient?

A. Dependent personality disorderB. Anxiety (avoidant) personality disorderC. Anankastic personality disorderD. Markedly accentuated personalityE. Psychopathy-like state

64. A 45-year-old man with thrombophlebitisof the deep veins in his legs suddenly afterphysical exertion developed sharp pain in histhorax on the right, dyspnea, and hemoptysis.Objectively his condition is severe; hepresents with acrocyanosis, shortening ofpulmonary percussion sound on the right,and weakened respiration. Respiration is30/min., blood pressure is 110/80 mm Hg.ECG shows sinus tachycardia, heart rate is120/min., electrical axis of the heart deviatesto the right, SI -QIII . What is the most likelydiagnosis?

A. Pulmonary embolismB. Community-acquired right-sidedpneumoniaC. Cancer of the right lungD. Right-sided exudative pleurisyE. Spontaneous pneumothorax

65. A 48-year-old woman has beenhospitalized due to development oftachysystolic atrial fibrillation. She has lost5 kg of body weight within 2 months. Onpalpation there is a node in the left lobe ofthe thyroid gland. What pathology resultedin the development of this condition?

A. Toxic nodular goiterB. Aterosclerotic cardiosclerosisC. Chronic thyroiditisD. Nontoxic nodular goiterE. Autoimmune thyroiditis

66. A 48-year-old woman developedinsomnia, depressive mood, anxiety, fears

Page 11: Krok 2 - Центр тестування при МОЗ УкраїниKrok 2 Medicine () Терапевтичний профiль 2 1. A 25-year-old woman has been suffering from diabetes

Терапевтичний профiль 11

and suicidal thoughts after the death ofher husband that occurred one month ago.During her stay in the hospital she speaksin a low voice, is depressed, anxious, avoidssleeping, refuses to eat. What medicationsshould be prescribed in this case?

A. AntidepressantsB. AntipsychoticsC. Group B vitaminsD. NootropicsE. Anticonvulsants

67. A 32-year-old woman complains ofepisodes of intense fear that occur withoutvisible cause and last for 10-20 minutes; theepisodes are characterized by rapid pulse,sweating, labored breathing, and vertigo.Specify the likely diagnosis:

A. Panic disorderB. Paranoid syndromeC. Manic syndromeD. Simple schizophreniaE. Claustrophobia

68. A 39-year-old man suffers fromchronic adrenal insufficiency and receivesreplacement glucocorticoid therapy(hydrocortisone - 15 mg/day). He is toundergo elective surgery for calculouscholecystitis. What medication adjustmentshould be made on the day of the surgeryto prevent the development of acute adrenalinsufficiency?

A. Increase the dosage by 2-3 timesB. Cancel the drug for the day of the surgeryC. Add a mineralocorticoidD. Add an antibioticE. Prescribe a large volume intravenous fluidinfusion

69. After a long drive with the windowopen a man developed facial asymmetry;he cannot close his right eye, his rightnasolabial fold is smoothed out, movementsof expression are absent on the right, thereis a disturbance of gustatory sensation in thetongue on the right. No other neurologicalpathologies were detected. What disease canbe provisionally diagnosed in this patient?

A. Neuropathy of the facial nerveB. Neuropathy of the trigeminal nerveC. Trigeminal ganglionitisD. Neuropathy of the oculomotor nerveE. Ischemic stroke

70. A 56-year-old woman was diagnosed withstage 2 hypertension of the 2nd degree. Shebelongs to the group of moderate risk andhas bronchial asthma. What group of drugs isCONTRAINDICATED to this patient?

A. β-blockersB. Angiotensin-converting enzyme inhibitorsC. DiureticsD. Calcium antagonistsE. Imidazoline receptor antagonists

71. A 45-year-old woman is registered forregular check-ups due to Werlhof disease(immune thrombocytopenia). Completeblood count: Нb- 100 g/L, erythrocytes -2.8 · 1012/L, platelets - 90.0 · 109/L, leukocytes- 8.4 · 109/L, erythrocyte sedimentation rate -13 mm/hour. Examination detects a singlesmall hematoma on the anterior surfaceof the thigh, developed after the patientaccidentally stumbled on a table. Whattreatment tactics should be chosen in thiscase?

A. Continue the supervision by the hospitalhematologistB. Urgent hospitalization into the hematologyunitC. Urgently start a hemostatic therapy,followed by a planned hospitalization into thehematology unitD. Urgent hospitalization into the generalcare unitE. Administer thrombocytic mass, continuethe treatment in the hematology unit

72. The dermatologist has an appointmentwith a 30-year-old man that complainsof severely itching rashes that especiallydisturb him at night. The rashes developed2 weeks ago, after he had returned from atravel. Objectively on the lateral surfacesof his fingers, hands, wrists, elbows, lowerabdomen, genitals, and thighs there arepaired papulovesicles, single pustules,and scratch marks. What disease can besuspected?

A. ScabiesB. PyodermaC. DermatitisD. EczemaE. Shingles

73. A 38-year-old woman developed amedical condition 7 days after her returnfrom Bangladesh. Periodical elevationof temperature was accompanied bychills and excessive sweating. She wasdiagnosed with tropical malaria. Next dayher condition further deteriorated: bodytemperature - 38oC , inertness, periodicalloss of consciousness, generalized seizures,tachycardia, hypotension, and icteric skin.What complication can be suspected in thiscase?

Page 12: Krok 2 - Центр тестування при МОЗ УкраїниKrok 2 Medicine () Терапевтичний профiль 2 1. A 25-year-old woman has been suffering from diabetes

Терапевтичний профiль 12

A. Cerebral comaB. Serous meningitisC. Purulent meningitisD. Acute hepatic failureE. Acute heart failure

74. A 73-year-old woman came to the familyphysician for one of her regular follow-upexaminations. Three months ago she wasfound to have type 2 diabetes mellitus.She was keeping to her diet and exerciseplan and taking phytopreparations. Onexamination her fasting glucose was withinthe range of 7.8-8.6 mmol/L, HbА1с - 7.9%.Height - 164 cm, weight - 83 kg. Whatblood sugar-controlling medicine shouldshe be prescribed first in the course of herpharmacological therapy?

A. MetforminB. GlibenclamideC. GlimepirideD. GliclazideE. Insulin

75. A 27-year-old man complains of pain inhis leg joints, purulent discharge from theeyes, and painful burning sensations duringurination. Disease onset was acute. He has ahistory of influenza. The patient smokes anddrinks alcohol in excess. In his line of workhe is often away on business trips. What is themost likely etiological factor of this disease?

A. ChlamydiaB. AdenovirusC. StreptococciD. StaphylococciE. Candida

76. A 46-year-old woman has diarrhea withabdominal distension, loss of body mass,and large amounts of porridge-like foul-smelling stool without blood streaks ortenesmus. Objective examination detectsmoderate tenderness in the mesogastriumand left abdominal flank. Feces analysisdetects steatorrhea with neutral fat and

creatorrhea. What prescription would be themost advisable in this case?

A. Multi-enzyme preparationsB. Cholinergic antagonistsC. Metronidazole and loperamideD. Antacids and antispasmodicsE. Cholinergic antagonists and antibacterialagents

77. A man was brought into the admissionroom after an overexposure to cold. Hecomplains of sharp pain in the small ofhis back and elevated body temperatureup to 38oC . He took some aspirin. Bloodtest: leukocytes - 10.5 · 1012/L, eosinophils- 5%, band neutrophils - 8%, segmentedneutrophils - 51%, lymphocytes - 32%,monocytes - 4%, erythrocyte sedimentationrate - 28 mm/hour. Urinalysis: protein - 0.6g/L, leukocytes - cover the whole vision field,large amount of mucus. What is the mostlikely diagnosis?

A. Acute pyelonephritisB. Chronic pyelonephritisC. Acute glomerulonephritisD. Tubulointerstitial nephritisE. Subacute malignant glomerulonephritis

78. A 26-year-old man complains ofchills, rhinitis, dry cough, and fever up to38oC . Examination shows him to be ina moderately severe condition; there aresmall pale pink non-merging spots on theskin of his back, abdomen, and extremities.Palpation reveals enlarged occipital andaxillary lymph nodes. No information aboutvaccination history could be obtained. Whatis the likely etiology of this disease?

A. Rubella virusB. Epstein-Barr virusC. StreptococcusD. Mumps virusE. Neisseria meningitis

Page 13: Krok 2 - Центр тестування при МОЗ УкраїниKrok 2 Medicine () Терапевтичний профiль 2 1. A 25-year-old woman has been suffering from diabetes

Хiрургiчний профiль 13

1. During medical examination a cadetin the naval college was detected to havea painless dense ulcer 1.5x0.5 in size in hisperianal area at the 2 o’clock position. Theulcer floor resembles ”old fat”. What is theprovisional diagnosis?

A. Hard syphilitic chancre of the rectumB. Rectal fissureC. Rectal fistulaD. Anal cancerE. Anal crypt suppuration

2. A 32-year-old woman complains of tumor-like formation on the anterior surface of herneck that appeared 2 years ago. Within thelast 3 months the tumor has been rapidlygrowing. It hinders swallowing and impairsspeech; the tumor causes a sensation ofpressure. Objectively the skin moisture isnormal, pulse is 80/min., rhythmic, bloodpressure is 130/80 mm Hg. In the right lobe ofthe thyroid gland there is a dense lumpy node3.0x3.5 cm that moves during swallowing.Scanning image shows a ”cold nodule” inthe thyroid gland. Make the provisionaldiagnosis:

A. Thyroid cancerB. Thyroid adenomaC. Thyroid cystD. Nodular goiterE. Autoimmune thyroiditis

3. After a surgery for a left thigh phlegmonthe disease progression was complicated bysepsis. On the 7th day after the surgerythere are marked signs of a generalizedinflammatory reaction, in blood there aresigns of toxic anemia and progressinghypoproteinemia, bilirubin levels are 40mcmol/L, AST and ALT exceed the normby 2.5 times. Oliguria persists (700 mL ofurine per day). Name the phase of sepsisprogression:

A. Catabolic phaseB. Stress phaseC. Anabolic phaseD. Recovery phaseE. Mixed phase

4. A 10-year-old boy, who was outdoors inwindy and cold weather, developed moderatepain and tingling in his fingers and toes.When he returned home, his parents noticedthat the tips of his fingers and toes were whiteand their sensitivity was lost. As the affectedareas were warming up, the fingers and toesdeveloped tingling and painful sensations.Skin pallor changed into redness, tinglingstopped, mild itching and swelling of thefingers appeared. Determine the frostbitedegree in this child:

A. Frostbite of the I degreeB. PerniosisC. Frostbite of the II degreeD. Frostbite of the III degreeE. Frostbite of the IV degree

5. A 16-year-old patient has made anappointment with an otolaryngologist. Hecomplains of elevated body temperatureand sore throat. Disease onset was 2 daysago, after the patient ate two portions ofice-cream. Pharyngoscopy shows hyperemicmucosa of the palatine tonsils, with purulentexudate in the lacunae. Make the provisionaldiagnosis:

A. Lacunar tonsillitisB. Follicular tonsillitisC. DiphtheriaD. Acute pharyngitisE. Pseudomembranous (Vincent’s) tonsillitis

6. A 35-year-old woman complains of highbody temperature and pain in the upperouter quadrant of her right buttock, whichdeveloped after an injection. She has beenpresenting with this condition for 3 days. Atthe site of injection the skin is hyperemic;there is a painful infiltrate with an areaof softening in its center. The woman isdiagnosed with a postinjection abscess ofthe right buttock. What tactics should thesurgeon choose in this case?

A. Abscess incision, sanation and drainage ofthe cavityB. Hospitalization, prescription of antibiotics,UHFC. Abscess puncture, pus removal followed byapplication of antisepticsD. 10-15 minutes of low-intensity laserradiation directed at the right buttockE. Antipyretic agents, massage, andapplication of dry heat to the right buttock

7. A 65-year-old woman on abdominalpalpation presents with a tumor in theumbilical region and above it; the tumor is13x8 cm in size, moderately painful, non-mobile, pulsing. On auscultation systolicmurmur can be observed. What is the mostlikely diagnosis?

A. Abdominal aortic aneurysmB. Gastric tumorC. Arteriovenous aneurysmD. Tricuspid insufficiencyE. Bicuspid insufficiency

8. A 32-year-old man complains of painin his legs that intensifies during walking,intermittent claudication, numbness of histoes, extremity coldness, and inability to walkmore that 100 meters. When he sleeps, hisleg usually hangs down. The patient has beensmoking since he was 16. He drinks alcoholin excess. The left leg is colder than the right

Page 14: Krok 2 - Центр тестування при МОЗ УкраїниKrok 2 Medicine () Терапевтичний профiль 2 1. A 25-year-old woman has been suffering from diabetes

Хiрургiчний профiль 14

one; the skin of the extremities is dry. Nopulse can be detected on the pedal arteries,while pulsation of the femoral arteries isretained. What is the most likely diagnosis?

A. Obliterating endarteritisB. Diabetic angiopathyC. Leriche syndrome (aortoiliac occlusivedisease)D. Raynaud diseaseE. Deep thrombophlebitis

9. A 50-year-old patient was brought toa hospital with complaints of blood inurine. Urination is painless and undisturbed.Macrohematuria had been observed for3 days. Objectively: kidneys cannot bepalpated, suprapubic area is withoutalterations, external genitalia are non-pathologic. On rectal investigation: prostateis not enlarged, painless, has normalstructure. Cystoscopy revealed no changes.What is the most likely diagnosis?

A. Renal carcinomaB. Bladder tuberculosisC. VaricoceleD. Dystopic kidneyE. Necrotic papillitis

10. A 59-year-old man complains of painin his left eye and left side of his head,significant vision impairment of the left eye,nausea, and vomiting. Visual acuity of theright eye is 1.0. Visual acuity of the lefteye is 0.03, attempts at correction bring noimprovement. Right eye intraocular pressure- 21 mm Hg, left eye intraocular pressure -65 mm Hg. Congestive injection is observedon the sclera of the left eye. The cornea isthick and swollen. The anterior chamber isshallow, moist, and clear. The pupil is dilatedand unresponsive to the light, the fundus ofthe eye is not visible. What is the most likelydiagnosis?

A. Acute attack of glaucoma of the left eyeB. Acute iridocyclitis of the left eyeC. Stage II intraocular tumor of the left eyeD. Endophthalmitis of the left eyeE. Panophthalmitis of the left eye

11. On the 15th day after a small traumaof the right foot, the patient developedindisposition, fatigability, irritability,headache, elevated body temperature, andsensation of constriction, tension, andtwitching in the muscles of the right shin.What disease can be suspected?

A. TetanusB. Anaerobic gas gangreneC. ErysipelasD. Acute thrombophlebitisE. Thrombophlebitis of the popliteal artery

12. A patient has the second and third degree

burns of the 15% of the body surface. On the20th day after the trauma the patient presentswith sharp increase of body temperature,general weakness, rapid vesicular respiration;facial features are sharpened, BP is 90/50 mmHg, heart rate is 112/min. What complicationis it?

A. SepsisB. PneumoniaC. Acute intoxicationD. Purulent bronchitisE. Anaerobic infection

13. 2 hours after a traffic accident a 28-year-old man in a grave condition was broughtto a hospital. The patient complains ofabdominal pain. He received a blow to theabdomen with the steering wheel. Objectiveexamination revealed the following: theabdomen does not participate in respiration,is tense and acutely painful on palpation;the abdominal muscles are defensivelytense, peritoneal irritation signs are positive,hepatic dullness is absent. BP is 90/60 mmHg, heart rate is 120/min. What furthertreatment tactics should be chosen?

A. LaparotomyB. LaparoscopyC. Cold to the abdomenD. Ultrasound investigationE. Laparocentesis

14. A 48-year-old woman has arrived tothe surgical unit with wounds in her thigh.On examination the wound surface hasdirty-gray coating with unpleasant sweetsmell. Wound content resembles raspberryjelly. Skin tissues around the wound areglossy and turgid. Palpation reveals moderatecrepitation in the tissues. What microflora isthe most likely to cause such inflammation?

A. Anaerobic clostridialB. Anaerobic non-clostridialC. StreptococciD. StaphylococciE. Blue pus bacillus

15. After a pain attack in the right subcostalarea, a 58-year-old woman with overnutritiondeveloped icteric skin and sclera, light-colored feces, and dark urine. Her abdomenis distended and painful on palpation in theright subcostal area. Palpation detects liverenlargement by 2-3 cm. Blood test: totalbilirubin - 90 mcmol/L, conjugated bilirubin- 60 mcmol/L. What method of examinationwill be the most informative for diagnosisclarification?

Page 15: Krok 2 - Центр тестування при МОЗ УкраїниKrok 2 Medicine () Терапевтичний профiль 2 1. A 25-year-old woman has been suffering from diabetes

Хiрургiчний профiль 15

A. Retrograde cholangiopancreatographyB. Intravenous cholegraphyC. Infusion cholegraphyD. Percutaneous transhepatic cholegraphyE. US of the hepatopancreatobiliary zone

16. An 11-year-old boy for a month has beenpresenting with increasing pain in the rightfemur. In the painful area there is a non-mobile painful tumor with unclear margins.The child complains of general indisposition,weakness, increased body temperature up to39oC . X-ray shows widened medullary cavity,small foci of cancellous bone destruction, andonion-like lamellar exfoliation of the corticallayer. What is the most likely pathologyresulting in such clinical presentation?

A. Ewing sarcomaB. Osteogenic sarcomaC. FibrosarcomaD. ChondrosarcomaE. Juxtacortical sarcoma

17. A 43-year-old man complains of aprotrusion in the right inguinal region,that enlarges due to strain. He has beenpresenting with this condition for 6 months.Within this period the protrusion has grown.Objectively in the right inguinal region anelastic protrusion 8x5 cm is visible. Onpalpation it disappears, leaving an emptyspace 4x4 cm between the pedicles ofthe Poupart ligament. ”Cough push” signis positive over this opening. Make thediagnosis:

A. Right-sided reducible inguinal herniaB. Right-sided reducible femoral herniaC. Cyst of the right spermatic cordD. Right-sided inguinal lymphadenitisE. Right-sided reducible arcuate line hernia

18. A 78-year-old man with a prostateadenoma underwent a herniotomy for adirect inguinal hernia. After the surgery hepresents with absent urination. Enlargedurinary bladder is detectable above thepatient’s pubis. What measures should betaken in this case?

A. Bladder catheterizationB. Apply cold to the urinary bladder areaC. Prescribe processing of the postoperativewound with UHF fieldD. Prescribe proserin (neostigmine)intramuscularlyE. Prescribe antispasmodics subcutaneously

19. A 38-year-old patient has been broughtby an ambulance to the surgical departmentwith complaints of general weakness,indisposition, black stool. On examinationthe patient is pale, there are dottedhemorrhages on the skin of his torso andextremities. On digital investigation there areblack feces on the glove. Blood test: Hb- 108

g/L, thrombocytopenia. Anamnesis statesthat a similar condition was observed 1 yearago. Make the diagnosis:

A. Thrombocytopenic purpuraB. HemophiliaC. Ulcerative bleedingD. Rectal tumorE. Nonspecific ulcerative colitis

20. A 30-year-old man came to the familyphysician. 2 months ago he underwent asurgery for open fracture of the humerus.On examination the patient’s condition issatisfactory; in the area of the postoperativewound there is a fistula that dischargesa small amount of pus; the area itself isred; fluctuation is detected. X-ray showsdestruction of the humerus with sequestra.What complication did the patient developduring the postoperative period?

A. Posttraumatic osteomyelitisB. Hematogenous osteomyelitisC. Wound suppurationD. Posttraumatic phlegmonE. Suture sinus

21. 3 hours after a trauma, a youngman developed bradycardia of 46/min.,anisocoria D>S, hemi-hyperreflexia S>D,hemihypesthesia on the left, and a convulsivedisorder. The character of this process needsto be clarified. What method of examinationwill be the most accurate for this purpose?

A. Brain CTB. Skull X-rayC. ElectroencephalographyD. EchoencephalographyE. Lumbar puncture

22. The body of a 24-year-old woman withsuspected poisoning has been found onthe street. Forensic medical examinationwas requested by an investigator duringexamination of the site and the body.According to the Criminal Procedure Codecurrently in force in Ukraine, forensicmedical examination is required when it isnecessary to determine the:

A. Cause of deathB. Manner of deathC. Time of deathD. Mode of deathE. Mechanism of death

23. A 37-year-old patient complains of painin the spinal column, reduced mobility. Thecondition persists for 7 years. ”Sway back”is observed, there is no movement in allspinal regions. X-ray shows ”bamboo spine”vertebral column. What is the most likelydiagnosis?

Page 16: Krok 2 - Центр тестування при МОЗ УкраїниKrok 2 Medicine () Терапевтичний профiль 2 1. A 25-year-old woman has been suffering from diabetes

Хiрургiчний профiль 16

A. Ankylosing spondyloarthritisB. OsteochondrosisC. Spondylitis deformansD. Tuberculous spondylitisE. Spondylolisthesis

24. A surgery unit received a person withan incised stab wound on the upper thirdof the right thigh. Examination detects anincised stab wound 3.0x0.5x2.0 cm in sizeon the inner surface of the upper third ofthe right thigh. Bright-red blood flows fromdeep within the wound in a pulsing stream.Characterize this type of bleeding:

A. ArterialB. VenousC. ParenchimatousD. CapillaryE. Mixed

25. A 47-year-old man developed thesigns of decompensated laryngeal stenosisagainst the background of acute flegmonouslaryngitis. He presents with inspiratorydyspnea at rest, forced position, cyanotic skincovered in cold sweat, tachycardia, deficientpulse, and low blood pressure. What urgenttreatment tactics should be chosen?

A. TracheostomyB. Oral administration of hyposensitizationsubstances and broncholyticsC. Intravenous administration of dehydratingagentsD. Administration of glucocorticoid hormonesE. Oxygen therapy

26. Heart X-ray of a 31-year-old man hasrevealed the following: with tightly filledopacified esophagus there is a marginal fillingdefect in its middle third on the posteriorwall; the defect is 1.8x1.3 cm in size withclear oval border. Mucosal folds are retainedand envelop the defect; wall peristalsisand elasticity are not affected. There areno complaints regarding the condition ofthe patient’s alimentary canal. Make theprovisional diagnosis:

A. Esophageal tumorB. Achalasia cardiaeC. Esophageal burnsD. DiverticulumE. Barrett esophagus

27. A 25-year-old man was hospitalized withcomplaints of pain in his lower abdomen andright lumbar area that appeared one hourago. Patient’s general state is moderatelysevere. Body temperature - 38.2oC , heart rate- 102/min. The tongue is dry. The abdomenis painful on deep palpation in the rightiliac area and in the Petit triangle. Aure-Rozanov and Gabay signs are positive. Makethe provisional diagnosis:

A. Acute appendicitisB. Right-sided renal colicC. Cecal tumorD. Intestinal obstructionE. Acute cholecystitis

28. A 45-year-old man diagnosed with acutepulmonary abscess suddenly developedsharp pain in his chest on the right anddyspnea up to 30/min. Examination detectsfacial cyanosis and shallow rapid respirations.Auscultation reveals acutely weakenedrespiration throughout the whole right lung;percussion reveals a vesiculotympanitic(bandbox) resonance at the lung apexand dullness in the lower lobe. Whatcomplication developed in this patient?

A. PyopneumothoraxB. PleuropneumoniaC. PneumothoraxD. Acute mediastinitisE. Esophageal perforation

29. A 5-year-old child was brought to theENT department by an ambulance. Thechild presents with cough and difficultrespiration. From the patient’s history itis known that the child was playing witha toy construction set, when suddenlystarted coughing and developed laboredbreathing. Examination detects periodicalcough, labored expiration, and respiratorylag in the left side of the child’s thorax.Auscultation: diminished respiration on theleft. Percussion: tympanitis. X-ray shows adisplacement of the mediastinal organs tothe right. Make the diagnosis:

A. A foreign body in the left bronchus,valvular bronchostenosisB. A foreign body in the right bronchus,valvular bronchostenosisC. A foreign body in the tracheaD. A foreign body in the left bronchus,complete bronchostenosisE. A foreign body in the right bronchus,partial bronchostenosis

30. A 30-year-old man was brought to theneurosurgical department with complaintsof constant headaches, nausea, vomiting,fever, and weakness of the right-side limbs.Anamnesis states that one month agothe patient had a surgery for left-sidedsuppurative otitis and mastoiditis. He hasbeen undergoing treatment in an ENTdepartment. Approximately 2 weeks agothe temperature increased, and the patientdeveloped headaches. Objectively: heart rate- 98/min., BP- 140/90 mm Hg, temperature -38.3oC . Neurologically manifested stiff neck:bilateral Kernig’s symptom, unsteadinessduring the Romberg’s maneuver. Computertomography of the brain revealed a three-dimensional growth with a capsule in the lefthemisphere. Make the diagnosis:

Page 17: Krok 2 - Центр тестування при МОЗ УкраїниKrok 2 Medicine () Терапевтичний профiль 2 1. A 25-year-old woman has been suffering from diabetes

Хiрургiчний профiль 17

A. Cerebral abscessB. EchinococcusC. HemorrhageD. HydrocephalusE. Arnold-Chiari malformation

31. The burns unit received a patient, who6 hours ago during a fire received flameburns. On the patient’s body there is gray-brown area of necrosis that covers 3/4 ofthe body perimeter. Occasionally there aresmall blisters with hemorrhagic contents andpatches of shredded epidermis. What localtherapy is necessary in this case?

A. Decompression necrectomyB. Chemical necrolysisC. Blister punctureD. Necrectomy with xenotransplantationE. Necrectomy with dermal autograft

32. A woman in her early- to mid-thirtieshas lost her consciousness 3-5 minutes ago.On examination: the skin is pale, no pulseover the carotid arteries, no spontaneousrespiration, pupils are dilated; the patientis nonresponsive, presents with atony. Thepatient’s condition can be determined as:

A. Clinical deathB. Natural deathC. SyncopeD. Brain deathE. Comatose state

33. A boy had a foreign body removedfrom under his nail plate. 3 days later hedeveloped a sharp throbbing pain at theend of his distal phalanx, which intensifieswhen the phalanx is pressed, hyperemia ofthe nail fold, elevated body temperature upto 38.5oC , and nail plate discoloration. Makethe diagnosis:

A. Subungual panaritiumB. ErysipelasC. ParonychiaD. ErysipeloidE. Abscess

34. A 32-year-old woman complains of bodyweight loss despite her increased appetite,nervousness, and tremor of the extremities.Objectively: the skin is moist; the thyroidgland is diffusely enlarged, painless, soft, andmobile. Blood test: increased level of T3,T4, and thyroid-stimulating hormone (THS).What is the most likely diagnosis?

A. Diffuse toxic goiterB. Thyroid carcinomaC. Autoimmune (Hashimoto’s) thyroiditisD. Thyroid adenomaE. Diffuse nontoxic goiter

35. A 19-year-old young man complainsof cough with expectoration of purulent

sputum in the amount of 100 mL perday, hemoptysis, dyspnea, increased bodytemperature up to 37.8oC , general weakness,weight loss. The patient’s condition lasts for4 years. Exacerbations occur 2-3 times ayear. The patient presents with malnutrition,pale skin, cyanosis of the lips, drumstick(clubbed) fingers. Tympanic percussionsound in the lungs, weakened respiration,numerous various moist crackles in the lowerpulmonary segments on the left can beobserved. In blood: erythrocytes - 3.2 ·1012/L,leukocytes - 8.4·109/L, ESR- 56 mm/hour. OnX-ray: lung fields are emphysematous, theleft pulmonary root is deformed and dilated.What is the most likely diagnosis?

A. Multiple bronchiectasis of the left lungB. Chronic left-sided pneumoniaC. Chronic abscess of the left lungD. Left-sided pulmonary cystic dysplasiaE. Suppuration of the cyst in the left lung

36. A 57-year-old woman during a regularultrasound examination presented with aspace-occupying heterogeneous lesion in theright kidney. What is the most informativemethod of renal tumor diagnostics?

A. Spiral computed tomographyB. Excretory urographyC. Retrograde pyelographyD. Radioisotope renographyE. Three glass urine test

37. A 40-year-old victim of a traffic accidentsustained the following injuries: closeddiaphyseal femur fracture, brain concussion,multiple rib fractures, hemopneumothorax,degloving shin injuries. What injuries requirethe most urgent attention?

A. Multiple rib fractures, hemopneumothoraxB. Closed diaphyseal femur fractureC. Brain concussionD. Degloving shin injuriesE. All injuries are equivalent

38. At the railroad crossing a passengertrain collided with a bus. In this collision 26bus passenges died, another 18 passengersreceived mechanical injuries of varyingseverity. Where will be professional medicalaid provided for the victims of this accident?Who will provide this aid?

A. In medico-prophylactic institutions;general physicians and surgeonsB. At the site of the accident; first-responseemergency teamsC. At the site of the accident; specializedsecond-response emergency teamsD. In medico-prophylactic institutions;specialized second-response emergency teamsE. In medical institutions; all listed types ofhealthcare workers

Page 18: Krok 2 - Центр тестування при МОЗ УкраїниKrok 2 Medicine () Терапевтичний профiль 2 1. A 25-year-old woman has been suffering from diabetes

Хiрургiчний профiль 18

39. A 45-year-old man underwent a cardiacsurgery one week ago. His general state hasbeen deteriorating since then: dyspnea atrest, retrosternal pain that irradiates to theneck, marked weakness. Objectively his bodytemperature is hectic. His cardiac bordersare expanded, apical beat is weakened.Auscultation detects pericardial friction rub.What is the most likely diagnosis?

A. Acute pericarditisB. Acute cardiac aneurysmC. Myocardial infarctionD. Acute myogenic dilatation of the heartE. Pulmonary embolism

40. A 45-year-old man was brought by an

ambulance into the emergency hospital. Hecomplains of sudden pain in the lumbar area,frequent painful urination, and vomiting.Examination detects pain in the lumbararea, costovertebral angle tenderness, painon palpation of kidneys and along theureter on the right. Urine test: proteins,fresh erythrocytes, leukocytes. Make theprovisional diagnosis:

A. Urolithiasis, renal colicB. Acute pyelonephritisC. Acute glomerulonephritisD. Acute renal failureE. Polycystic kidney disease

Page 19: Krok 2 - Центр тестування при МОЗ УкраїниKrok 2 Medicine () Терапевтичний профiль 2 1. A 25-year-old woman has been suffering from diabetes

Педiатричний профiль 19

1. A newborn girl has Apgar score of 7-8points at the 1-5 minutes after birth. Duringthe labor there was a brief difficulty withextraction of the shoulder girdle. After birththe baby presents with disturbed function ofthe proximal segment and forced positionof the right arm. The shoulder is rotatedinwards, the elbow is extended, the forearmis pronated, and the whole upper limbresembles an arm of a doll. What is the mostlikely clinical diagnosis in this case?

A. Erb-Duchenne palsyB. Thoracic spine traumaC. Osteomyelitis of the right armD. Intracranial hemorrhageE. Soft tissue injury of the right arm

2. Disease onset was acute. A childdeveloped general weakness, pain in thejoints, and elevated temperature. Later thesesigns became accompanied by itching skinrash manifested as erythematous spots 2-5 mm in size. The rash gradually turnedhemorrhagic. Large joints are painful andswollen; pain attacks periodically occur inthe paraumbilical area; there are signs ofintestinal hemorrhage. What is the mostlikely diagnosis?

A. Hemorrhagic vasculitis (Henoch-Schonleinpurpura)B. Scarlet feverC. Hemorrhagic meningoencephalitisD. Streptococcal impetigoE. Rheumatism

3. A 13-year-old girl for the last two weekshas been complaining of dyspnea andshin and foot edemas that appear after aphysical exertion. In the morning the edemassignificantly decrease. Clinical examinationrevealed enlarged liver and coarse systolicmurmur over the heart area. Blood test andurinalysis are without changes. What is themost likely cause of edemas in this child?

A. Heart failureB. Nephrotic syndromeC. Acute pyelonephritisD. Angioneurotic edemaE. Hepatic cirrhosis

4. A 7-year-old boy has severe pulmonarymucoviscidosis (cystic fibrosis). Hecomplains of dyspnea and bloodexpectoration. Objectively he presents withlagging physical development, acrocyanosis,hepatomegaly, drumstick fingers, and nailplates resembling a ”clock face”. Provisionaldiagnosis of chronic pulmonary heart diseaseis made. What examination would be themost informative for diagnosis confirmation?

A. Doppler echocardiographyB. ElectrocardiographyC. Chest X-rayD. Rheography of the pulmonary arteryE. Ultrasound of the liver

5. Mother of a 5-year-old child noticed on thethe head of her child a round ”bald” spot 3cm in diameter. All the hairs in the focus arebroken off at the length of 5-6 mm. The daybefore the child was petting a stray cat. Makethe diagnosis:

A. MicrosporiaB. Superficial trichophytosisC. Deep trichophytosisD. PsoriasisE. Alopecia areata

6. A 2-year-old child with persisting coughand subfebrile body temperature after a caseof URTI developed dyspnea, cyanosis ofthe nasolabial triangle, percussion dullnessand weakened respiration in the lower lobeof the right lung, and a slight mediastinaldisplacement to the left. What pulmonarypathology is likely to cause this clinicalpresentation?

A. PleurisyB. EmphysemaC. PneumoniaD. AtelectasisE. Bronchitis

7. During examination a 4-month-oldchild with meningococcemia presents withacrocyanosis, cold extremities, tachypnea,and thready pulse, blood pressure of 30/0mm Hg, anuria, and sopor. What clinicalsyndrome is it?

A. Toxic shock syndromeB. NeurotoxicosisC. ExicosisD. Encephalic syndromeE. Acute renal failure

8. At night a 2-year-old child withupper respiratory tract infection suddenlydeveloped dyspnea with labored inspiration.Objectively the skin is pale, perioral cyanosisand slight acrocyanosis are observed.Breathing is loud, respiration rate is 32/min.Jugular, supra- and infraclavicular fossaeretract during breathing. Respiration iscoarse on auscultation. Heart sounds areclear and sonorous, heart rate is 120/min.What condition was complicated by thedevelopment of the upper respiratory tractinfection?

A. Stenosing laryngotracheitisB. Airway foreign bodyC. Obstructive bronchitisD. BronchiolitisE. Bronchial asthma

Page 20: Krok 2 - Центр тестування при МОЗ УкраїниKrok 2 Medicine () Терапевтичний профiль 2 1. A 25-year-old woman has been suffering from diabetes

Педiатричний профiль 20

9. A 1-year-old child with a case of URTIsuddenly developed noisy respirations withdifficult inspiration, intercostal retractions,and barking cough on the 2nd night afterthe disease onset. What is the most likelydiagnosis?

A. Stenosing laryngotracheobronchitisB. Acute pulmonary inflammationC. Bronchial asthmaD. Acute bronchitisE. Acute bronchiolitis

10. A 10-year-old boy with symptoms ofarthritis and myocarditis was brought to ahospital. Based on clinical examination theprovisional diagnosis of juvenile rheumatoidarthritis was made. What symptom is themost contributive for the diagnostics of thisdisease?

A. Reduced mobility of the joints in themorningB. Regional hyperemia of the jointsC. Affection of the large jointsD. Enlarged heartE. Increased heart rate

11. A 7-year-old girl has been twice treatedwith antibacterial agents for urinary tractinfection. US shows no severe renal defects.The child presents with recurrence ofleukocyturia and bacteriuria, elevated bodytemperature up to 38.5oC , and pain in her leftlumbar area. What examination should beconducted first to clarify the cause of urinaryinfection recurrence?

A. Micturating cystourethrographyB. Excretory urographyC. Retrograde pyelographyD. ImmunogramE. Radioisotope renography

12. A child is 1 year old. After solid foodwas introduced into the diet, within thelast several months the child developedloss of appetite, diarrhea with large amountof feces, and occasional vomiting. Bodytemperature remains normal. Body weightis 7 kg. The child is very pale, has leg edemasand extremely distended abdomen. Fecesanalysis detects high levels of fatty acids andsoaps. Diagnosis of celiac disease was madeand gluten-free diet was prescribed. Whatshould be excluded from the diet in this case?

A. Cereals - wheat, oatsB. Milk and dairy productsC. FruitsD. Animal proteinE. Easily digestible carbohydrates

13. A 7-year-old boy has been an inpatientfor 1.5 months. He had been brought tothe hospital with complaints of edemasall over his body, low urine output, and

headache. Clinical urinalysis: proteins -7.1 g/L, leukocytes - 1-2 in the visionfield, erythrocytes - 3-4 in the vision field.During the course of treatment the edemasgradually dissipated, headache abated,diuresis normalized. Daily urine proteins -3 g/L. Biochemical blood test: total protein -43.2 g/L, urea - 5.2 mmol/L, cholesterol - 9.2mmol/L. What glomerulonephritis syndromeis the most likely to be present in the patient?

A. NephroticB. NephriticC. Isolated urinaryD. HematuricE. Mixed

14. A 3-month-old child with signs of ricketspresents with positive Chvostek, Trousseau,and Maslov signs. One day ago the parentswitnessed a cyanotic attack in their child -the child broke into a cold sweat, the eyesbulged, and respiratory arrest occurred. Oneminute later the child drew in a loud breathand the child’s condition normalized again.What is the cause of the described signs ofthe disease?

A. Decrease of blood calcium levelsB. Increase of blood calcium levelsC. Decrease of blood phosphorus levelsD. Increase of blood phosphorus levelsE. Metabolic acidosis

15. A newborn with gestational age of31 weeks presents with hypotonia anddepressed consciousness. Hematocrit is35%, general cerebrospinal fluid analysisshows increased content of erythrocytesand protein, and low glucose. These datacorrespond with the clinical presentation of:

A. Intracranial hemorrhageB. MeningitisC. SepsisD. AnemiaE. Intrauterine infection

16. A newborn has Apgar score of 9. Whenshould this infant be put to the breast?

A. In the delivery roomB. After 12 hoursC. After 2 hoursD. On the 2nd dayE. On the 3rd day

17. A 3-week-old infant developed large,flaccid vesicles with purulent contents onthe skin of chest and abdomen. The vesiclesrupture quickly. Make the provisionaldiagnosis:

Page 21: Krok 2 - Центр тестування при МОЗ УкраїниKrok 2 Medicine () Терапевтичний профiль 2 1. A 25-year-old woman has been suffering from diabetes

Педiатричний профiль 21

A. Pemphigus neonatorumB. VesiculopustulosisC. Toxic erythemaD. Pemphigus syphiliticusE. Pseudofurunculosis

18. 10 hours after birth a child developedjaundice, hypotonia, hyporeflexia, andmoderate hepatosplenomegaly. Feces andurine are of normal color. Umbilical cordblood bilirubin is 51 mcmol/L due tounconjugated bilirubin levels. In venousblood: erythrocytes - 3.5 ·1012/L, Нb- 140 g/L,reticulocytes - 1.5%, bilirubin - 111 mcmol/L,conjugated - 11 mcmol/L, ALT- 40 U/L, AST-30 U/L. Mother’s blood group is А(II) Rh(-), child’s blood group is А(II) Rh(+). Whatlaboratory test can confirm the diagnosis?

A. Coombs testB. Viral hepatitis markers analysisC. Measurement of erythrocyte osmoticresistanceD. ErythrocytometryE. Measurement of glucose 6-phosphatedehydrogenase levels in erythrocytes

19. A 6-month-old child on breastfeedingis hospitalized in the inpatient department.After the child recovers, the doctorrecommends the mother to start introducingsolid food to the child’s diet. What productsshould be introduced to the child’s diet first?

A. Vegetable pureeB. Fermented dairy productsC. Grated appleD. Semolina porridgeE. Buckwheat porridge

20. The 5-year-old child has been ill for 2weeks. Cough attacks developed first andwere then followed by reprises. Duringcoughing the child’s face turns red andcervical veins bulge. The cough attacksinduce vomiting. X-ray shows intensifiedbronchial pattern. Blood test: leukocytes -16 · 109/L , lymphocytes - 72%, erythrocytesedimentation rate - 4 mm/hour. What is themost likely diagnosis?

A. PertussisB. Obstructive bronchitisC. PneumoniaD. Adenovirus infectionE. Foreign body

21. A 3-year-old child presents withdyspnea that abates in the sitting position,occasional loss of consciousness and seizures,delayed physical development, cyanosis,drumstick fingers. Echocardioscopy detectsaortic dextraposition, ventricular septaldefect, pulmonary artery stenosis, and rightventricular hypertrophy. What is the mostlikely diagnosis?

A. Tetrad of FallotB. Coarctation of the aortaC. Transposition of the great vesselsD. Ventricular septal defectE. Acquired valvular disease

22. A 15-year-old girl complains of dizzinessand sensation of lack of air that she developsin emotionally straining situations. Reliefoccurs after she takes corvalol. Objectively:hyperhidrosis and marble-like pattern ofthe skin of her palms and feet. Clinicaland instrumental examination revealed noorganic changes in the central nervous,cardiovascular, and respiratory systems.What provisional diagnosis can be made?

A. Somatoform autonomic dysfunctionB. Obstructive bronchitisC. Bronchial asthmaD. Stenosing laryngotracheitisE. Acute epiglottitis

23. A 1.5-month-old child on breasfeedingpresents from birth with daily vomiting,irregular liquid foamy feces, and meteorism,which are resistant to antibacterial andprobiotic therapy; no increase of body massis observed. The child’s condition improved,when breastmilk was substituted with ”NANlow lactose” formula. What pathology is it?

A. Lactase deficiencyB. Intestinal lambliasis (Giardiasis)C. Infectious enteritisD. Drug-induced enteritisE. Functional dyspepsia

24. A 13-year-old girl for a month has beencomplaining of fatigability, dull pain in herright subcostal area, abdominal distension,and constipations. Abdominal palpationreveals positive Kehr, Murphy, and Ortnersigns, while Desjardins and Mayo-Robsonpoints are painless. Total bilirubin is 14.7mcmol/L, predominantly indirect, ALT- 20U/L, AST- 40 U/L, amylase - 6.3 mmol/L.Echocholecystography shows practically nocontraction of the gallbladder. Make theprovisional diagnosis:

A. Hypokinetic biliary dyskinesiaB. Hyperkinetic biliary dyskinesiaC. Chronic pancreatitisD. Acute pancreatitisE. Chronic hepatitis

25. A 22-day-old infant developedsubcutaneous red nodes from 1.0 to 1.5 cm insize on the scalp; later the nodes suppurated.Temperature increased up to 37.7oC ,intoxication symptoms appeared, regionallymph nodes enlarged. Complete bloodcount: anemia, leukocytosis, neutrocytosis,increased ESR. What diagnosis can be made?

Page 22: Krok 2 - Центр тестування при МОЗ УкраїниKrok 2 Medicine () Терапевтичний профiль 2 1. A 25-year-old woman has been suffering from diabetes

Педiатричний профiль 22

A. PseudofurunculosisB. PemphigusC. VesiculopustulosisD. Scalp phlegmonE. -

26. A 10-year-old boy was brought intothe hospital with complaints of expiratorydyspnea, respirations are 30/min. He explainshis state by a change in the weatherconditions. For the last 4 years the boy hasbeen registered for regular check-ups dueto his diagnosis of third degree persistentbronchial asthma. To provide emergency aidfor this child, first he needs to be given:

A. Salbutamol or short-acting β2-agonistsB. DexamethasoneC. AdrenalineD. Euphylline (Aminophylline)E. Claritin (Loratadine)

27. A 3-year-old child has been brought toa hospital with complaints of pain in thelegs, fever, and loss of appetite. Objectively:pale skin and mucosa, hemorrhagic rash.Lymph nodes are enlarged, painless, denseand elastic, not matted together. Bones,joints, and abdomen are painful. The liverand spleen are enlarged. Hemogram: Hb- 88g/L, color index - 1.3, platelets - 80 · 109/L,leukocytes - 25.8 · 109/L, lymphoblasts - 70%,ESR- 52 mm/hour. Make the provisionaldiagnosis:

A. Acute leukemiaB. Thrombocytopenic purpuraC. Acute rheumatic feverD. Infectious mononucleosisE. Hemorrhagic vasculitis (Henoch-Schonleinpurpura)

28. During an outdoors school event inhot weather, a 10-year-old girl lost herconsciousness. Body temperature - 36.7oC .

Objectively her skin is pale and cold to touch,her pupils are dilated. Blood pressure - 90/50mm Hg. Heart rate - 58/min. What pathologyoccurred in this case?

A. SyncopeB. Sympathicotonic collapseC. Paralytic collapseD. SunstrokeE. -

29. A 13-year-old girl has 30% of excessivebody mass, she started to gain weight atthe age of 3. She has a family history ofobesity. Her height and sexual developmentare normal for her age. The appetite isexcessive. She complains of periodicalheadaches. Blood pressure - 120/80 mm Hg.Subcutaneous fat is evenly distributed, shehas no stretch marks. There is juvenile acneon her face. What type of obesity is it?

A. Alimentary constitutive obesityB. Hypothalamic obesityC. Adrenal obesityD. Hypothalamic syndrome of pubertyE. Hypothyroid obesity

30. An 8-year-old girl complains of frequentpainful urination in small amounts andurinary incontinence. The signs havebeen present for 2 days already. Sheexplains her disease by overexposure tocold. Costovertebral angle tenderness isabsent. Complete blood count is withoutpathologies. Urine test: leukocytes - 20-30in the vision field, erythrocytes - 40-50 in thevision field, unchanged, bacteriuria. What isthe most likely diagnosis?

A. CystitisB. VulvitisC. PyelonephritisD. GlomerulonephritisE. Urolithiasis

Page 23: Krok 2 - Центр тестування при МОЗ УкраїниKrok 2 Medicine () Терапевтичний профiль 2 1. A 25-year-old woman has been suffering from diabetes

Акушерство i гiнекологiя 23

1. A pregnant woman is 28 years old.Anamnesis: accelerated labor complicatedby the II degree cervical rupture. Thefollowing two pregnancies resulted inspontaneous abortions at the terms of 12 and14 weeks. On mirror examination: the uterinecervix is scarred from previous ruptures at9 and 3 hours, the cervical canal is gaping.On vaginal examination: the cervix is 2 cmlong, the external orifice is open 1 cm wide,the internal orifice is half-open; the uterus isenlarged to the 12th week of pregnancy, soft,mobile, painless, the appendages are withoutchanges. What diagnosis can be made?

A. Isthmico-cervical insufficiency, habitualnoncarrying of pregnancyB. Threatened spontaneous abortionC. Incipient abortion, habitual noncarrying ofpregnancyD. Cervical hysteromyoma, habitualnoncarrying of pregnancyE. Cervical pregnancy, 12 weeks

2. On the day 4 after the cesarean sectiona woman developed fever with bodytemperature up to 39oC and abdominalpain. Pulse - 104/min. She vomited twice.The patient is sluggish, her tongue is dryand has gray coating. The abdomen isdistended. Signs of peritoneal irritation arepositive in all segments. Peristalsis cannotbe auscultated. No passage of gas occurs.Uterine fundus is located at the level of thenavel. The uterus is painful on palpation. Thedischarge is moderate and contains bloodand pus. What is the most likely diagnosis?

A. Diffuse peritonitisB. MetroendometritisC. Progressive thrombophlebitisD. Pelvic peritonitisE. Parametritis

3. A 58-year-old woman came to thegynecological clinic. She complains ofbloody discharge from her genital tracts.Menopause is 8 years. Gynecologicalexamination: the uterus is slightly enlarged,dense to touch, with limited mobility; theuterine appendages cannot be detected;parametrium is free. Fractional curettage ofthe uterine cavity yields a significant amountof medullary substance in the scrape. What isthe most likely diagnosis?

A. Uterine corpus cancerB. AdenomyosisC. ChorioepitheliomaD. Uterine cervix cancerE. Hormone-producing ovarian tumor

4. A 48-year-old woman complains ofdisturbed menstrual cycle: her periods lastfor 7-9 days and are excessively profusethroughout the last half-year. She notesoccasional hot flashes in her head, insomnia,

irritability, and headaches. Her skin isof normal color. Blood pressure - 150/90mm Hg, pulse - 90/min., rhythmic. Theabdomen is soft and painless. Bimanualexamination shows no uterine enlargement,the appendages cannot be detected. Thevaginal fornices are free. What is the mostlikely diagnosis?

A. Climacteric syndromeB. Premenstrual syndromeC. Adrenogenital syndromeD. Stein-Leventhal syndrome (polycysticovary syndrome)E. Uterine myoma

5. A 30-year-old multigravida has beenin labour for 18 hours. 2 hours ago thepushing stage began. Fetal heart rate is clear,rhythmic, 136/min. Vaginal examinationreveals complete cervical dilatation, the fetalhead in the pelvic outlet plane. Sagittalsuture is in line with obstetric conjugate,the occipital fontanel is near the pubis. Thepatient has been diagnosed with primaryuterine inertia. What is the further tacticsof labor management?

A. Outlet forcepsB. Labour stimulationC. Cesarean sectionD. Skin-head Ivanov’s forcepsE. Vacuum extraction of the fetus

6. A woman is 40 weeks pregnant.The fetus is in the longitudinal lie andcephalic presentation. Pelvic size: 26-29-31-20. Expected weight of the fetus is 4800 gram.The labor contractions has been lasting for12 hours, within the last 2 hours they wereextremely painful, the parturient womanis anxious. The waters broke 4 hours ago.On external examination the contractionring is located 2 finger widths above thenavel, Henkel-Vasten sign is positive. Fetalheart rate is 160/min., muffled. On internalexamination the uterine cervix is fully open,the head is engaged and pressed to theentrance into the lesser pelvis. What is themost likely diagnosis?

A. Threatened uterine ruptureB. Complete uterine ruptureC. Hyperactive uterine contractionsD. Abruption of the normally positionedplacentaE. Anatomically contracted pelvis

7. A 23-year-old woman came to thegynecological clinic. She complains of pain,itching, and burning in her vulva, generalweakness, indisposition, elevated bodytemperature up to 37.2oC , and headache. Onexamination in the vulva there are multiplevesicles up to 2-3 mm in diameter withclear contents against the background ofhyperemia and mucosal edema. Make the

Page 24: Krok 2 - Центр тестування при МОЗ УкраїниKrok 2 Medicine () Терапевтичний профiль 2 1. A 25-year-old woman has been suffering from diabetes

Акушерство i гiнекологiя 24

provisional diagnosis:

A. Genital herpes infectionB. Primary syphilisC. Papillomavirus infectionD. Vulvar cancerE. Cytomegalovirus infection

8. A woman with the pregnancy term of 8weeks complains of elevated temperature upto 37.6oC , skin rash that can be characterizedas macular exanthema, enlargement ofposterior cervical and occipital lymphnodes, small amount of bloody dischargefrom the genital tracts. She was examinedby the infectious diseases specialist anddiagnosed with rubella. What tactics shouldthe obstetrician-gynecologist choose?

A. AbortionB. Prescription of antibacterial therapyC. Prescription of antiviral therapyD. Treatment of incipient abortionE. Prescription of hemostatic therapy

9. A 16-year-old girl has primaryamenorrhea, no pubic hair growth, normallydeveloped mammary glands; her genotype is46 ХY; uterus and vagina are absent. What isyour diagnosis?

A. Testicular feminization syndromeB. Mayer-Rokitansky-Kuster-HausersyndromeC. Cushing syndromeD. Sheehan syndromeE. Cushing disease

10. A 46-year-old woman came to thematernity clinic with complaints of moderateblood discharge from the vagina, whichdeveloped after the menstruation delay of 1.5months. On vaginal examination: the cervixis clean; the uterus is not enlarged, mobile,painless; appendages without changes. Makethe diagnosis:

A. Dysfunctional uterine bleedingB. AdenomyosisC. Ectopic pregnancyD. Submucous uterine myomaE. Cancer of the uterine body

11. A 25-year-old woman was brought intothe gynecological department with profusebloody discharge from her genital tracts.She is 12 weeks pregnant, the pregnancyis planned. Within the last 3 days she wasexperiencing pains in her lower abdomenthat eventually started resembling cramps,she developed bleeding. Her skin is pale,pulse - 88/min., blood pressure - 100/60 mmHg, body temperature - 36.8oC . Vaginalexamination: the uterus size correspondswith 11 weeks of pregnancy, the cervicalcanal allows inserting 1 finger and containsfragments of the fertilized ovum, the

discharge is bloody and profuse. What is themost likely diagnosis?

A. 12-week pregnancy, spontaneous abortionin progressB. 12-week pregnancy, threatenedspontaneous abortionC. Disturbed menstrual cycle,hyperpolymenorrheaD. Disturbed menstrual cycle, amenorrheaE. Full-term pregnancy, term labor

12. A 30-year-old woman came to thegynecological department. She complainsof sharp pain in her lower abdomen andtemperature of 38.8oC . She has a history ofextramarital sexual activity and 2 artificialabortions. On gynecological examinationthe uterus is unchanged. The appendagesare bilaterally enlarged and painful. Profusepurulent discharge is being produced fromthe vagina. What examination needs to beconducted to clarify the diagnosis?

A. Bacteriological and bacterioscopic analysisB. HysteroscopyC. Curettage of the uterine cavityD. ColposcopyE. Laparoscopy

13. It is the 3rd day after the first normalterm labor; the infant is rooming-in with themother and is on breastfeeding. Objectively:the mother’s general condition is satisfactory.Temperature is 36.4oC , heart rate is 80/min.,BP is 120/80 mm Hg. Mammary glandsare soft and painless; lactation is moderate,unrestricted milk flow. The uterus is dense,the uterine fundus is located 3 finger widthsbelow the navel. Lochia are sanguino-serous,moderate in volume. Assess the dynamics ofuterine involution:

A. Physiological involutionB. SubinvolutionC. LochiometraD. Pathologic involutionE. Hematometra

14. A 28-year-old woman complaining ofirregular menstruations and infertility cameto the gynecological clinic. Menstruationsoccur since the age of 15, irregular, withdelays up to 2 months. On examination shepresents with marked hirsutism and excessivebody weight. On vaginal examination theuterus is reduced in size and painless.The ovaries on the both sides are denseand enlarged. Ultasound shows microcysticchanges in the ovaries, the ovaries are5х4 cm and 4.5х4 cm in size with denseovarian capsule. Basal body temperatureis monophasic. What is the most likelydiagnosis?

Page 25: Krok 2 - Центр тестування при МОЗ УкраїниKrok 2 Medicine () Терапевтичний профiль 2 1. A 25-year-old woman has been suffering from diabetes

Акушерство i гiнекологiя 25

A. Polycystic ovary syndromeB. Krukenberg tumorC. Endometrioid cystsD. Bilateral adnexitisE. Ovarian carcinoma

15. An 18-year-old girl was brought into thegynecology deparment with complaints ofelevated body temperature up to 37.8oC ,sharp pain in her lower abdomen, moreintense on the right, and difficult defecation.Vaginal examination detected a painfuldense elastic formation 5x6 cm in the areaof her right ovary. Pregnancy test is negative.What is the most likely diagnosis?

A. Torsion of ovarian tumor pedicleB. Ectopic pregnancyC. AppendicitisD. Ovarian cyst ruptureE. Ovarian apoplexy

16. A 26-year-old woman presents withamenorrhea. 10 months ago she gave birthfor a second time. In her early postpartumperiod she developed a massive hypotonichemorrhage. No breasfeeding. Lately shehas been presenting with loss of weight,loss of hair, and indisposition. Gynecologicalexamination revealed atrophy of the externalgenitals, the uterus is abnormally small, nouterine appendages can be detected. What isthe most likely diagnosis?

A. Sheehan syndrome (postpartum pituitarygland necrosis)B. Physiological amenorrheaC. Suspected progressing ectopic pregnancyD. Stein-Leventhal syndrome (polycysticovary syndrome)E. Galactorrhea-amenorrhea syndrome

17. A 45-year-old woman came to thematernity clinic with complaints of periodicalpains in her mammary glands that start1 day before menstruation and stop afterthe menstruation begins. Palpation of themammary glands detects diffuse nodespredominantly in the upper outer quadrants.What is the most likely diagnosis?

A. Fibrocystic mastopathyB. Breast cancerC. MastitisD. HyperprolactinemiaE. Breast cyst

18. A 14-year-old girl came to the generalpractitioner with complaints of weakness,loss of appetite, headache, rapid fatigability.Her last menstruation was profuse and lastedfor 14 days after the previous delay of 2months. Objectively: the skin is pale, heartrate is 90/min., BP is 110/70 mm Hg, Hbis 88 g/L. Rectal examination: the uterusand its appendages are without changes,no discharge from the genital tracts. What

complication occurred in the patient?

A. Posthemorrhagic anemiaB. Somatoform autonomic dysfunction ofhypotonic typeC. MigraineD. GastritisE. Dysmenorrhea

19. A 22-year-old postparturient woman onthe 12th day after the normal childbirthinforms of elevated body temperature up to39oC for the last 3 days and pain in her rightmammary gland. The right mammary glandis enlarged, hot to touch, tense, hyperemic,and painful. Palpation reveals there a denseinfiltration 8x8 cm with a fluctuation in itscenter. What is the most likely diagnosis?

A. Postpartum period, day 12. Right-sidedinfiltrative-purulent mastitisB. Postpartum period, day 12. Right-sidedserous mastitisC. Postpartum period, day 12. Right-sidedgangrenous mastitisD. Postpartum period, day 12. Right-sidedphlegmonous mastitisE. Postpartum period, day 12. Right-sidedlactostasis

20. A 35-year-old pregnant woman withdegree 1 essential hypertension, developededemas and headache at the 33 week of herpregnancy. Objectively her general conditionis satisfactory, blood pressure - 160/100 mmHg, normal uterine tone. Fetal heart rateis 140/min., rhythmic. She was diagnosedwith daily proteinuria - 4 g/L, daily diuresis- 1100 mL. Creatinine - 80 mcmol/L, urea- 7 mmol/L, platelets - 100 · 109/L. Whatcomplication of pregnancy occurred?

A. Moderate preeclampsiaB. Severe preeclampsiaC. Mild preeclampsiaD. Hypertensive crisisE. Renal failure

21. A 24-year-old pregnant woman onher 37th week of pregnancy has beenbrought to the maternity obstetric servicewith complaints of weak fetal movements.Fetal heartbeats are 95/min. On vaginalexamination the uterine cervix is tiltedbackwards, 2 cm long, external orifice allowsinserting a fingertip. Biophysical profile ofthe fetus equals 4 points. What tactics ofpregnancy management should be chosen?

Page 26: Krok 2 - Центр тестування при МОЗ УкраїниKrok 2 Medicine () Терапевтичний профiль 2 1. A 25-year-old woman has been suffering from diabetes

Акушерство i гiнекологiя 26

A. Urgent delivery via a cesarean sectionB. Treatment of placental dysfunction andrepeated analysis of the fetal biophysicalprofile on the next dayC. Doppler measurement of blood velocity inthe umbilical arteryD. Urgent preparation of the uterine cervixfor deliveryE. Treatment of fetal distress; if ineffective,then elective cesarean section on the next day

22. During regular preventive gynecologicalexamination a 30-year-old woman wasdetected to have dark blue punctulated”perforations” on the vaginal portion ofthe uterine cervix. The doctor suspectsendometriosis of the vaginal portion of theuterine cervix. What investigation methodwould be most informative for diagnosisconfirmation?

A. Colposcopy, target biopsy of the cervixB. US of the lesser pelvisC. HysteroscopyD. Curettage of the uterine cavityE. Hormone testing

23. A parturient woman is 30 years old, stageI of the labor is ongoing. The fetus is inthe cephalic presentation. Auscultation ofthe fetal heart sounds detects bradycardia.Evaluation of cardiotocogram yielded thefollowing data: decrease of basal heart ratedown to 90/min., variability - monotonous (2and less); late decelerations with amplitudeof 50/min. Make the diagnosis and choosethe obstetrical tactics necessary in this case:

A. Fetal distress. Urgent cesarean sectiondeliveryB. Fetal distress. Vacuum extraction deliveryC. Normal condition of the fetus. VaginalbirthD. Fetal distress. Stimulation of uterinecontractionsE. Fetal distress. Forceps delivery

24. A 27-year-old woman complains offoul-smelling discharge from her genitaltracts, pain in her lower abdomen, andelevated temperature. The complaints arose2 days ago. She has a history of surgicalabortion at the term of 8 weeks one weekago. Mirror examination: the uterine cervixis clear, external orifice produces foul-smelling discharge. Vaginal examination: theuterus lies in anteflexion, is mobile, painful,and slightly enlarged. The appendages arewithout changes. Make the provisionaldiagnosis:

A. Postabortal endometritisB. EnterocolitisC. AppendicitisD. Acute respiratory diseaseE. Salpingoophoritis

25. A 17-year-old girl has made anappointment with the doctor. She plans tobegin her sex life. No signs of gynecologicalpathology were detected. In the familyhistory the patient’s grandmother hadcervical cancer. The patient was consultedabout the maintenance of her reproductivehealth. What recommendation will be themost helpful for prevention of invasivecervical cancer?

A. Vaccination against human papillomavirus(HPV)B. Vitamins, calcium, omega-3C. ImmunomodulatorsD. Antiviral and antibacterial drugsE. Timely treatment of sexually transmitteddiseases

Page 27: Krok 2 - Центр тестування при МОЗ УкраїниKrok 2 Medicine () Терапевтичний профiль 2 1. A 25-year-old woman has been suffering from diabetes

Гiгiєна та органiзацiя охорони здоров’я 27

1. A 38-year-old woman works in flaxprocessing, she dries flax. She came to thehospital complaining of difficult breathing,constricting sensation in her chest, and coughattacks. These signs appear on the first day ofher working week and gradually diminish onthe following days. What respiratory diseaseis likely in this case?

A. ByssinosisB. SilicosisC. Allergic rhinopharyngitisD. Bronchial asthmaE. Asthmatic bronchitis

2. A district doctor has diagnosed one ofhis patients with dysentery. What accountingdocument reflects this type of morbidity?

A. Urgent reportB. Statistical reportC. Report on a major non-epidemic diseaseD. Certificate of temporary disabilityE. Control card of a patient registered forregular check-ups

3. A 39-year-old man, a battery attendant,suddenly developed weakness, loss ofappetite, nonlocalized colicky abdominalpains, and nausea. Objectively his skin isgray; there is a pink-gray stripe on his gums;the stomach is soft and sharply painful. Bloodtest detected erythrocytes with basophilicstippling and anemia. The patient has ahistory of peptic ulcer disease of the stomach.Constipation tends to occur every 3-4days. What is the most likely provisionaldiagnosis?

A. Saturnism (lead poisosning)B. Acute appendicitisC. Perforation of gastric ulcerD. Acute cholecystitisE. Chronic alcoholism

4. A 9-month-old infant presents withdelayed tooth eruption and fontanel closure,weakness, and excessive sweating. What typeof hypovitaminosis is the most likely in thischild?

A. Hypovitaminosis DB. Hypovitaminosis CC. Hypovitaminosis B1D. Hypovitaminosis B6E. Hypovitaminosis A

5. A 10-year-old girl exhibits high level ofphysical development (� + 3σ), her bodylength increased by 10 cm within a year(which is double the norm for her age group),the number of permanent teeth correspondswith the age norm (20), the developmentof her secondary sex characteristics isthree years ahead of her age (Ма, Р, Ах,Menarche). Development rate ahead of herbiological age can occur due to:

A. Endocrine disordersB. AccelerationC. Certain components of her dietD. Sports trainingE. Deficient hygienic education

6. Employees work in conditions of highdust concentration. Certain chemical (silicondioxide content) and physical properties ofdust aerosols contribute to the developmentof occupational dust-induced diseases. Whatis the main physical property of dustaerosols?

A. DispersionB. MagnetizationC. Electric chargeD. SolubilityE. Ionization

7. In the factory cafeteria there wasan outbreak of food poisoning. Clinicalpresentation indicates staphylococcaletiology of this disease. 15 people are sick.To confirm the diagnosis of food poisoning,samples need to be sent to the laboratory.What samples should be obtained foranalysis?

A. Vomit massesB. Blood for hemocultureC. Blood (complete blood count)D. UrineE. Saliva

8. In April during the medical examinationof various population groups, 27% ofindividuals presented with low workingability and rapid fatigability. The followingsymptoms were observed in the affectedindividuals: swollen friable gingiva thatbleeds when pressed, hyperkeratosisfollicularis not accompanied by skin dryness.These symptoms most likely result from thefollowing pathology:

A. C-hypovitaminosisB. ParodontosisC. A-hypovitaminosisD. B1-hypovitaminosisE. Polyhypovitaminosis

9. Increased general morbidity of thelocal population is observed in the areanear a factory, where atmosphere is beingintensively polluted with sulfurous gas. Whateffect does polluted air have on human bodyin this case?

A. Chronic nonspecificB. Acute specificC. Acute nonspecificD. Chronic specificE. Selective

10. During medical examination of high and

Page 28: Krok 2 - Центр тестування при МОЗ УкраїниKrok 2 Medicine () Терапевтичний профiль 2 1. A 25-year-old woman has been suffering from diabetes

Гiгiєна та органiзацiя охорони здоров’я 28

middle school students, the doctors vereassessing correlation between biological andcalendar age of the school students basedon the following criteria: height growth rateper year, ossification of the carpal bones, thenumber of permanent teeth. What additionaldevelopment criterion should be assessed atthis age?

A. Development of secondary sexcharacteristicsB. Body massC. Chest circumferenceD. Vital capacity of lungsE. Hand strength

11. During analysis of morbidity in thecity, it was determined that age structureof population is different in each district.What statistical method allows to excludethis factor, so that it would not skew themorbidity data?

A. StandardizationB. Wilcoxon signed-rank testC. Correlation-regression analysisD. Dynamic time series analysisE. Analysis of average values

12. Clinical statistical investigation wasperformed to determine effectiveness ofa new pharmacological preparation forpatients with ischemic heart disease. Whatparametric test (coefficient) can be used toestimate the reliability of the results?

A. Student’s t-distributionB. Sign testC. Matching factorD. Wilcoxon signed-rank testE. Kolmogorov-Smirnov test

13. In a rural health care area there isan increasing cervical cancer morbidityobserved. The decision is made to conducta medical examination of the women livingin this locality. What type of medicalexamination is it?

A. TargetB. PreliminaryC. RegularD. ComplexE. Screening

14. In the process of hiring, a prospectiveemployee has undergone preventive medicalexamination and was declared fit to work inthis manufacturing environment. What typeof preventive medical examination was it?

A. PreliminaryB. ScheduledC. PeriodicalD. SpecificE. Comprehensive

15. On laboratory investigation of a pork

sample there is 1 dead trichinella detectedin 24 sections. This meat should be:

A. Sent for technical disposalB. Allowed for sale with no restrictionsC. Processed and sold through public cateringnetworkD. Processed for boiled sausage productionE. Frozen until the temperature of -10oC isreached in the deep layers, with subsequentexposure to cold for 15 days

16. To assess the effectiveness of medicaltechnologies and determine the power anddirection of their effect on the public healthindicators, the research was conducted tostudy the immunization rate of children andmeasles incidence rate by district. Whatmethod of statistical analysis should beapplied in this case?

A. Calculation of correlation coefficientB. Calculation of morbidity index among thenonvaccinatedC. Calculation of matching factorD. Calculation of standardized ratioE. Calculation of statistical significance of thedifference between two estimates

17. Having studied the relationship betweenthe distance from villages to the localoutpatient clinics and frequency of visits tothe clinics among the rural population ofthis area, it was determined that the rankcorrelation coefficient in this case equals -0.9.How can this relationship be characterized?

A. Strong inverse relationshipB. Strong direct relationshipC. Moderate inverse relationshipD. Moderate direct relationshipE. -

18. In the inpatient gynecological unit withina year 6500 women underwent treatment.They spent there a total of 102000 bed-days.What indicator of the gynecological unitwork can be calculated based on these data?

A. Average length of inpatient stayB. Average bed occupancy rate per yearC. Number of beds by hospital departmentD. Bed turnover rateE. Planned bed occupancy rate per year

19. A middle school teacher with 4-year-long record of work was issued a medicalcertificate for pregnancy and childbirth leave.What amount of pay will she receive for theduration of her leave in this case?

A. 100% of average salaryB. 50% of average salaryC. 70% of average salaryD. 60% of average salaryE. 80% of average salary

20. In the air of the feed kitchen at the

Page 29: Krok 2 - Центр тестування при МОЗ УкраїниKrok 2 Medicine () Терапевтичний профiль 2 1. A 25-year-old woman has been suffering from diabetes

Гiгiєна та органiзацiя охорони здоров’я 29

poultry factory, at the area where formulafeed is being mixed, the dust concentrationreaches 200 mg/m3. Air microflora isrepresented predominantly by Asperqillusand Mucor fungi. What effect determinespathogenic properties of the dust?

A. AllergenicB. TeratogenicC. MutagenicD. FibrogenicE. Toxic

21. During regular medical examination alyceum student presents with signs of cheilitisthat manifests as epithelial maceration inthe area of lip seal. The lips are bright-red, with single vertical cracks covered withbrown-red scabs. These clinical signs aremost likely caused by insufficient content ofthe following in the diet:

A. RiboflavinB. Ascorbic acidC. RetinolD. ThiamineE. Calciferol

22. A 30-year-old woman made anappointment with the family doctor forscheduled vaccination of her 2-year-oldchild. What type of healthcare provides suchmedical services?

A. Primary healthcareB. Emergency aidC. Secondary healthcareD. Tertiary healthcareE. Palliative care

23. Human body receives from theatmosphere a number of chemicals. Whattype of action results in the combined effectthat is less than the sum of isolated effects ofthese chemicals on the body?

A. AntagonismB. PotentiationC. Isolated actionD. Synergistic actionE. Complex action

24. Clinical trials have proved the”Lipoflavon” drug to be effective fortreatment of unstable angina pectoris in thecontrol group and experimental group ofpatients. Neither patients nor researchersknew who belonged to which group. Namethis type of study:

A. Double blind studyB. Simple blind studyC. Triple-blind studyD. Total-blind studyE. Multicenter study